You are on page 1of 86

ACKNOWLEDGEMENT

The time and effort provided by the following individuals who served as members of this committee are
greatly appreciated:

Kimberly Cavanagh, DHSc, MPAS, PA-C, Chair


Jonathan Bowser, MS, PA-C
Petar Breitinger, MPAS, PA-C
Christine Bruce, MHSA, PA-C
Denise Dillingham, MPAS, PA-C
Kathleen Ehrhardt, MMS, PA-C
Rex Hobbs, MPAS, PA-C
Annette Larson, MSPAP, PA-C
Eric Vangsnes, PhD, MS, PA-C

Subcommittee Members
Alan Gindoff, DHSc, MPAS, PA-C
Ryan Junsay, MSPA, PA-C
Susan King-Barry, BSN, BSM, MPAS, PA-C
Marc Maller, MD

DEDICATION

This examination would not have been possible without the years of commitment of the MR. TIB
Development Committee. Numerous PA educators from across the nation provided their experience and
insight as questions for MR. TIB. It has been this data bank that served as the building blocks for
PACKRAT.

PAEA is proud to be able to continue in the tradition of quality fostered by the forerunners of the self-
assessment examination for physical assistants. It is our honor to dedicate PACKRAT to:

Jesse C. Edwards, MS
Claire S. Parker, PhD
University of Nebraska, Physician Assistant Program

Copyright 2009. Physician Assistant Education Association 1


PHYSICIAN ASSISTANT EDUCATION ASSOCIATION

Physician Assistant Clinical Knowledge Rating and Assessment Tool (PACKRAT)

Form 14

Directions and Explanations

TABLE OF CONTENTS

I. Introduction 1

II. Explanation of the Score Report 2

Page 1: Scores Your total Score and Group Comparisons 2


Page 2: Your STRENGTHS, Weaknesses, and Quality of Responses 2
Page 3: Your Individual and Correct Responses 2
Page 4: Your Responses by Task and Category 2
Page 5-6: Your Profile Comparison: Demographic Profile 2

III. Recommendations for Using the Feedback Package 3

IV. Study Resources 5

V. Answer Key 7

VI. Examination Explanations 8

VII. Comment Form 82

Copyright 2009. Physician Assistant Education Association. All rights reserved. No part of this publication may be reproduced or
transmitted in any form or by any means, electronic or mechanical, including photocopy or recording, or any information and
retrieval system, without permission in writing from the Physician Assistant Education Association.

Copyright 2009. Physician Assistant Education Association 2


PHYSICIAN ASSISTANT CLINICAL KNOWLEDGE RATING
AND ASSESSMENT TOOL (PACKRAT)

I. Introduction

The Physician Assistant Clinical Knowledge Rating and Assessment Tool (PACKRAT) was developed by
a volunteer committee of experts and is based on the content outline of a nationally recognized
competency examination. The following is a description of the content of PACKRAT:

PACKRAT EXAMINATION MATRIX


PACKRAT EXAMINATION MATRIX
CONTENT AREA NUMBER OF ITEMS
CONTENT AREA NUMBER OF ITEMS
1. History & Physical 36
1. History & Physical
2. Diagnostic Studies 26 32
2. Diagnostic Studies
3. Diagnosis 36 41
3. Diagnosis
4. Health Maintenance 45 22
4. Health Maintenance
5. Clinical Intervention 22 31
5. Clinical Intervention
6. Clinical Therapeutics 32 41
6. Clinical Therapeutics
7. Scientific Concepts 48 22
7. Scientific Concepts 16
TOTALS 225
TOTALS 225

Additionally, questions also apply to the following clinical specialties:


Additionally, questions also apply to the following clinical specialties:
A. Cardiology J. Obstetrics/Gynecology
A. Cardiology
B. Dermatology K. I.Orthopedics/Rheumatology
Neurology
B. Dermatology
C. Endocrinology M.J.Psychiatry/Behavioral
Obstetrics/GynecologyMedicine
C. Endocrinology
F. Gastrointestinal/Nutritional N. K. Orthopedics/Rheumatology
Pulmonology
D. ENTH. Hematology P. L. Pediatrics
Urology/Renal
E. Ophthalmology
I. Neurology Q.M. Psychiatry/Behavioral
Infectious Diseases Medicine
F. Gastrointestinal/Nutritional N. Pulmonology
G. Geriatrics O. Surgery
H. Hematology P. Urology/Renal

The task and specialty categories for each item are listed in the answer key on page 5; your feedback
package contains a breakdown of responses by the task and clinical specialty category. Pay particular
attention to the questions you answered incorrectly and determine the specialty for that question and use
this information to identify weaknesses.

The PACKRAT provides a detailed feedback report of performance and it is available to anyone at any
time. Explanations were developed for all the questions to provide a rationale for correct, as well as
incorrect, answers. This information will help determine strengths and weaknesses with respect to the
PACKRAT content outline. If you have weaknesses in specific areas, you may need to obtain additional
clinical experience in those areas.

Copyright 2009. Physician Assistant Education Association 3


This booklet is designed to explain and interpret the information contained in the accompanying
computerized score report. You can use the report package to learn more about your abilities.
Page 3 lists your answers to all questions.
II. Explanation of the Score Report When your answer differs from the correct
one, the proper response appears in
This section provides an interpretation of each parentheses. Use Section VI with this page to
page of the computerized score report you review the rationale for each option that is
received. You should have your computer provided in the explanations, which are
score report in front of you. Begin on page 1 referenced to the study resources. The
of the report and read the following explanations may help you understand why
information. one answer is more appropriate than another,
or not the best answer, and why some of your
Page 1: Scores Your Total Score and answers may have been incorrect. If the
Group Comparisons option you chose was judged potentially
harmful to the patient or others, an asterisk (*)
Page 1 is an overview of the PACKRAT appears before your answer. Options
feedback report. Toward the bottom of the classified as potentially harmful may identify
page is your examination score. This score serious weaknesses. Go over these questions
shows the number of questions you answered carefully and read the explanations for the
correctly out of a possible 225. The average correct answers. You may be able to identify
score for all first-year and second-year areas where you need further study.
candidates who have taken the PACKRAT to
date is also given. Page 4: Your Responses by Task and
Specialty Category
Page 2: Your Strengths, Weaknesses, and
Quality of Responses Page 4 lists your responses by both specialty
and task category. You will be able to identify
Page 2 of the score report gives an overview the areas of the content outline where you
of the content area in which your performance may have difficulty. The numbers reflect how
is categorized as Strong, Satisfactory, or many items you answered correctly out of the
Needing Improvement. These areas are total possible correct within each task and
based on the examination matrix on page 1. specialty area. Categories 1 through 7 identify
the task areas and A-P the clinical specialty
In each content area, your answers have areas. If you missed a significant number of
been classified as correct, acceptable, items in an area, check the key and go over
unsatisfactory, or harmful. A definition of the explanations for the items in these areas.
these classifications is also provided on this
page. Pay particular attention to the areas Page 5-6: Your Profile Comparison:
under Needing Improvement, as these areas Demographic Profile
should be noted for further study. Also check
the answer key for the specialty area of these Page 5 is the beginning of the Demographic
items. If you selected a harmful answer in any Profile Comparison for programs. This profile
content area, it will be automatically placed in shows the reported demographic information
the Needing Improvement category, and compares your students information to
regardless of the number of correct answers the entire group of individuals who have taken
selected. Carefully review these questions the PACKRAT to date. The demographic data
and their explanations and specialty are based on the information your students
classifications in Section VI to help you provided when answering the demographics
understand why your answers were correct. questions. The summary demographic
information shown reflects all the data
compiled for either first or second-year
Page 3: Your Individual and Correct students who taken the PACKRAT.
Responses
RESPONDENT DEMOGRAPHIC
INFORMATION

Copyright 2009. Physician Assistant Education Association 4


future certification examinations. Should you
C. Number of months of clinical rotations wish to provide suggestions about PACKRAT
completed at the time of this exam? to PAEA, you will find a comment form on the
last page of this booklet.
Your response: All Second-year
Respondents: IV. Study Resources

( ) 1. None ( 0% ) A variety of textbooks are currently available


( ) 2. Less than 3 months ( 5% ) to assist candidates in preparing for the
( ) 3. 3 to 6 months ( 15%) certification examination. For additional
( ) 4. 7 to 9 months ( 20%) information, you may contact a faculty
( ) 5. 10 to 12 months (45%) member at an educational program or an
( ) 6. Greater than 12 months (15%) experienced colleague if you need help
determining which references to review in a
specific content area. A short list of general
Programs can utilize this information in order textbooks is below. All examination questions
to compare the characteristics of their first are related to material found in these
year students and second year students to the resources.
national demographics of first year and
second year students who have taken the Please note that the books on this list are not
PACKRAT test. available from PAEA. This is not intended as
an all-inclusive list, and the materials listed
III. Recommendations for using The below are suggested study materials only.
Feedback Package
1. Andreoli TE, et al (eds). Cecils
As a current physician assistant student, Essentials of Medicine. 7th ed.
PACKRAT can be a useful self-evaluation Philadelphia, PA: WB Saunders,
tool. Through careful review of question an Elsevier company, 2007.
explanations, noting specific tasks and 2. Ballweg R et al. Physician
content areas, you will be able to assess your Assistant: A Guide to Clinical
current strengths and weaknesses. You will Practice. 4th ed., WB Saunders,
be able to identify particular areas in which to an Elsevier company, 2008.
concentrate more effort as you continue your 3. Bickley, LS. Bates Guide to
studies. By concentrating your effort on the Physical Examination and History
areas in which you did not do well, you may Taking. 9th ed. Philadelphia, PA:
improve your performance, and you may have JB Lippincott Co., 2007.
a better chance of passing the proctored 4. Beckmann CR, et al. Obstetrics &
examination. However, PAEA cannot Gynecology. 5th ed. Philadelphia,
guarantee that this will occur, since the PA: Lippincott Williams & Wilkins,
conditions under which you attempted the 2006.
PACKRAT may have been different from 5. Behrman RE, et al. Nelsons
those in a standardized administration of a Textbook of Pediatrics. 18th ed.
proctored examination. Philadelphia, PA: WB Saunders,
Use the explanations in Section VI to analyze an Elsevier company, 2007.
why you chose various options. Again, pay 6. Berkowitz, C. Pediatrics: A
particular attention to the options that were Primary Care Approach, 3rd ed.
judged potentially harmful or unsatisfactory. Philadelphia, PA: WB Saunders,
Look at the question and the four options an Elsevier company 2008.
again to see why the answer you chose was 7. Fauci AS, et al (eds). Harrisons
incorrect. If there appears to be a deficit in Principles of Internal Medicine.
your exposure to a particular clinical specialty, 17th ed. New York, NY: McGraw-
perhaps further study would make you more Hill, Inc., 2008.
familiar with these situations. 8. DeCherney AH & Pernoll ML
Once you have completely reviewed your (eds.) Current Obstetric &
score report and this booklet, PAEA hopes Gynecological Diagnosis &
you will use this information to improve your Treatment, 10th ed., Norwalk, CT:
overall performance, either on the job or on Appleton & Lange, 2006

Copyright 2009. Physician Assistant Education Association 5


9. Ellsworth AJ, et al. (eds). Mosbys 22. Noble J, et al. Textbook of
Medical Drug Reference. Primary Care Medicine. 3rd ed.
Philadelphia, PA: Mosby, 2007. Philadephia PA: Mosby, 2001. (In
10. Wolf K, Johnson RA, and PAEA library 2/07)
Surmond D. Fitzpatricks Color 23. Riordan-Eva, P. and Whitcher, JP.
Atlas and Synopsis of Clinical Vaughn & Asburys General
Dermatology. 5th ed. New York, Ophthalmology. 17th ed., McGraw
NY: McGraw-Hill, Inc., 2005. Hill, 2007.
11. Goldman J and Bennet JC. Cecil 24. Sacher RA and McPherson RA.
Textbook of Medicine. 23rd ed. Widmann's Clinical Interpretation
Philadelphia, PA: WB Saunders. of Laboratory Tests. 11th ed. FA
2008. Davis Co., 2000. (In PAEA library
12. Goroll, AH, Mulley, AG, & May, 2/07)
LA. Primary Care Medicine: Office 25. Schwartz SI, et al. Principles of
Evaluation and Management of Surgery. 8th ed. New York, NY:
the Adult Patient. 6th ed., McGraw-Hill, Inc., 2004.
Lippincott Williams &Wilkins, 2009 26. Skinner HB (ed.) Current
(To be released). Diagnosis & Treatment in
13. Hay WW, et al. Current Pediatric Orthopedics. 4th ed.,
Diagnosis and Treatment. 19th Norwalk,CT:Appleton & Lange,
ed. Norwalk, CT: McGraw Hill, 2008.
2008. 27. Steinberg GG. Orthopedics in
14. Kaplan HI and Sadock BJ (eds). Primary Care. 3rd ed.
Synopsis of Psychiatry. 10th ed., Philadelphia, PA: Lippincott
Philadelphia, PA: Williams & Williams & Wilkins, 3rd ed, 1999.
Wilkins, 2007. 28. Tierney LM, et al. Current Medical
15. Katzung BG. Basic and Clinical Diagnosis and Treatment. 48th
Pharmacology. 10th ed. ed. Stamford, CT: Appleton &
Stamford, CT: Appleton & Lange, Lange, 2009.
2006. 29. Tintinalli JE, Kelen GD, and
16. Mandel GL, Bennett JE, and Dolin Stapezynski JS. Emergency
R. Principles and Practice of Medicine: A Comprehensive
Infectious Disease. 6th ed., Study Guide. 6th ed. New York,
Philadelphia, PA: Churchill NY: McGraw-Hill, Inc., 2004.
Livingston, an Elsevier company, 30. Townsend CM. Sabistons
2005. ISBN#0443066434 Textbook of Surgery. The
17. Marx, J., Hockberger, RS, and Biological Basis of Modern
Walls, RM. Rosens Emergency Surgical Practice. 18 ed.
Medicine: Concepts and Clinical Philadelphia, PA: WB Saunders,
Practice. 6th ed., Philadelphia, PA: an Elsevier company, 2008.
Mosby, 2005. 31. Doherty GM. Current Surgical
18. McPhee SJ, et al. Diagnosis and Treatment. 12th
Pathophysiology of Disease. 5th ed., McGraw Hill, 2005.
ed., McGraw Hill, 2005. 32. Wilson WR. Current Diagnosis
19. Mercier LR, et al. Practical and Treatment in Infectious
Orthopedics. 6th ed. Philadelphia, Disease. Norwalk, CT: Appleton
PA: Mosby, an Elsevier company, & Lange, 2001.
2008. ISBN#9780323036184
20. Mettler FA, et al. Primary Care
Radiology. Philadelphia, PA: WB
Saunders, Co., 2000. (In PAEA
library 2/07)
21. Howland RD nad Mycek MJ.
Lippincotts Illustrated Reviews:
Pharmacology. 4th ed.
Baltimore, MD: Williams & Wilkins,
2008.

Copyright 2009. Physician Assistant Education Association 6


THIS PAGE INTENTIONALLY
LEFT BLANK FOR ANSWER
KEY

Copyright 2009. Physician Assistant Education Association 7


PHYSICIAN ASSISTANT EDUCATION ASSOCIATION

Physician Assistant Clinical Knowledge Rating and Assessment Tool

(PACKRAT) Form 14

EXPLANATIONS

1. History & Physical/Cardiology


When performing a pre-participation sports physical in the adolescent population, a murmur with which of the
following qualities indicates a risk for sudden death during exercise?
A. Increases with the Valsalva maneuver
B. Increases with squatting maneuver
C. Associated with a mid-systolic click
D. Mid-systolic without radiation to the carotids
Explanations
(c) A. Hypertrophic cardiomyopathy (HCM) is a known cause of sudden death during or just after physical exertion
and competitive sports. The murmur associated with HCM is worsened by conditions that cause reduced
ventricular volume such as the Valsalva maneuver, sudden standing, and tachycardia.
(u) B. Typical systolic flow murmurs will become accentuated with maneuvers which increase venous blood flow to
the heart and these murmurs do not place athletes at risk for sudden cardiac death.
(u) C. Mitral valve prolapse is the most common type of heart murmur that is associated with a mid-systolic click.
Mitral valve prolapse does not place the patient at risk for sudden cardiac death.
(u) D. A mid-systolic heart murmur that fails to radiate into the carotids is most commonly associated with a benign
systolic flow murmur and does not place the athlete at risk for sudden cardiac death.
Ref: (7)

2. Scientific Concepts/Obstetrics/Gynecology
At the time of ovulation in a normal menstrual cycle, there is a peak in the serum concentration of which of the
following?
A. Luteinizing hormone
B. Prostaglandin
C. Progesterone
D. Prolactin
Explanations
(c) A. Luteinizing hormone is responsible for ovulation and, therefore, peaks at that time.
(u) B. Prostaglandin is likely associated with the production of vasospasm, vascular necrosis, and menstrual flow, not
ovulation.
(u) C. The majority of progesterone is secreted by the corpus luteum and, therefore, peaks after ovulation has
occurred.
(u) D. Prolactin is an anterior pituitary hormone, and although important in reproduction and pregnancy, it is not
present in high levels at the time of ovulation.
Ref: (8)

8
3. Diagnostic Studies/Hematology
A positive direct Coombs' test may be seen in which of the following conditions?
A. G6PD deficiency
B. Sickle cell anemia
C. Hereditary spherocytosis
D. Autoimmune hemolytic anemia
Explanations
(u) A. See D for explanation.
(u) B. See D for explanation.
(u) C. See D for explanation.
(c) D. A positive direct Coombs' test indicates that antibody has attached to an antigen on the RBC which causes
agglutination. Autoimmune hemolytic anemia results when the patient has antibodies against their own RBCs.
G6PD deficiency does result from an antibody antigen reaction. Sickle cell anemia is caused by a defective
hemoglobin which detected by hemoglobin electrophoresis. Hereditary spherocytosis is detected utilizing the
osmotic fragility test.
Ref: (28)

4. Health Maintenance/Urology/Renal
In order to prevent the progression of diabetic nephropathy which of the following medications should be instituted?
A. Lisinopril (Prinipril)
B. Propanolol (Inderal)
C. Verapamil (Calan)
D. Hydrochlorothiazide (Diuril)
Explanations
(c) A. All patients should be started on an ACE inhibitor to prevent the progression of proteinuria. ACE inhibitors
appear to improve glomerular hemodynamics by decreasing glomerular pressure.
(u) B. Beta blockers are not indicated for the treatment of microalbuminuria.
(u) C. Calcium channel blockers are not indicated for the treatment of microalubuminuria.
(u) D. Thiazide diuretics are not indicated for the treatment of microalbuminuria.
Ref: (28)

5. Diagnosis/Pulmonology
A 5 year-old male presents with a history of recurrent episodes of acute bronchitis, characterized by fever and
productive cough. He has no known significant past medical history. His pulmonary examination reveals crackles in
the bilateral lower lobes. The remainder of his physical examination is normal. Chest x-ray demonstrates platelike
atelectasis and dilated, thickened airways in the middle and lower lungs. Which of the following is the most likely
diagnosis?
A. Acute bronchitis
B. Bronchiectasis
C. Pneumonia
D. Tuberculosis
Explanations
(u) A. Barring underlying pulmonary pathology, the chest x-ray in acute bronchitis should be normal.
(c) B. Bronchiectasis typically presents as recurrent episodes of acute bronchitis. Platelike atelectasis and dilated
and thickened airways, sometimes described as tram lines, are common radiographic findings.
(u) C. While the history may suggest pneumonia, the radiographic findings do not support this diagnosis.
(u) D. Tuberculosis would present with cavitating granuloma formation more commonly at the apices.
Ref: (5)

9
6. History & Physical/Orthopedics/Rheumatology
Which of the following clinical characteristics is associated with bicipital tendonitis?
A. Aggravated by resisted supination of the forearm
B. Bulging appearance to the proximal arm
C. Weakness of the arm with internal rotation and adduction
D. Pain that awakens the patient at night
Explanations
(c) A. Supraspinatus tendonitis will be aggravated by resisted supination of the forearm.
(u) B. Biceps rupture may present with a bulging appearance of the proximal arm.
(u) C. Weakness of the arm with internal rotation and adduction is characteristic of pectoralis major rupture or tear.
(u) D. Night pain is characteristic of rotator cuff tear or tendonitis.
Ref: (26)

7. Clinical Therapeutics/Cardiology
Which of the following antiarrhythmic drugs can be associated with hyper- or hypothyroidism following long-term use?
A. Quinidine
B. Amiodarone
C. Digoxin
D. Verapamil
Explanations
(u) A. See B for explanation.
(c) B. Amiodarone is structurally related to thyroxine and contains iodine, which can induce a hyper- or hypothyroid
state.
(u) C. See B for explanation.
(u) D. See B for explanation.
Ref: (7)

8. Diagnostic Studies/Neurology
An 18 year-old male is involved in a motor vehicle accident with a question of cervical spine fracture. What is the
imaging test of choice to initially evaluate this patient and clear his cervical c-spine?
A. Positron emission tomography
B. Magnetic resonance imaging
C. Computed tomography
D. Lateral radiograph
Explanations
(u) A. There is no role for positron emission tomography in suspected cervical spine injury.
(u) B. MRI and CT of the spine may be performed in the setting of acute cervical spine injury when a major fracture
or dislocation is identified.
(u) C. See B for explanation.
(c) D. Cervical spine x-rays are most commonly used as the initial screen for cervical spine injury. A cervical spine
series consists of a lateral view, anteroposterior (AP) view, and an odontoid view. The lateral view detects up
to 80% of traumatic spine injuries.
Ref: (29)

10
9. Clinical Intervention/ENT/Ophthalmology
A 4 year-old boy presents with pain and irritation of his left ear. Otoscopic examination reveals an insect in the left
auditory canal. The tympanic membrane is not completely visualized. Which of the following is the most appropriate
management of this patient?
A. Debrox insertion with suction removal
B. Irrigation with room temperature saline
C. Insertion of 2% lidocaine solution with suction or forceps removal
D. Polymyxin drop insertion via wick
Explanations
(u) A. Debrox is used for cerumen impaction not foreign body removal.
(u) B. Irrigation with room temperature saline is useful for small particle removal only if the tympanic membrane is
well-visualized and without perforation. It is not indicated in the removal of an insect.
(c) C. Two percent lidocaine solution will paralyze the insect and provide topical anesthesia for suction or forceps
removal.
(u) D. Polymyxin B is indicated in otitis externa and administered via a wick when there is significant edema of the
auditory canal. It is not indicated in the removal of a foreign body.
Ref: (29)

10. Diagnosis/Gastrointestinal/Nutritional
A 62 year-old male is brought to the emergency department with acute hematemesis. The patient denies a previous
history of vomiting. His wife states he has chronic liver disease. Physical examination reveals a distended abdomen
without rebound, guarding or organomegaly. There is a fluid wave. Which of the following is the most likely
diagnosis?
A. Esophageal varices
B. Mallory-Weiss tear
C. Arteriovenous malformation
D. Perforated duodenal ulcer
Explanations
(c) A. Esophageal varices are dilated submucosal veins that develop in a patient with underlying portal
hypertension. The most common cause of portal hypertension is cirrhosis.
(u) B. A patient with a Mallory-Weiss tear would have a history of retching but would not have a distended
abdomen.
(u) C. Most arteriovenous malformations are asymptomatic. If symptomatic they would have symptoms of a slow
bleed.
(u) D. A patient with perforated duodenal ulcer would have rebound and guarding on examination.
Ref: (1)

11. Scientific Concepts/Dermatology


Which of the following is characterized by epidermal hyperplasia and an increase in the epidermal turnover?
A. Atopic dermatitis
B. Tinea corporis
C. Ecthyma
D. Psoriasis
Explanations
(u) A. Inflammation in atopic dermatitis results in lichenification of the skin from itching, which leaves the skin dry and
scaly when it resolves. The inflammation is the result of elevated T-lymphocyte activation, defective cell-
mediated immunity, and IgE overproduction. Epidermal hyperplasia is not involved in atopic dermatitis.
(u) B. Tinea corporis is a fungal infection and does not have epidermal hyperplasia.
(u) C. Ecythma is the result of an infection from group A beta-hemolytic streptococcus and contaminated with
staphylococci. Like atopic dermatitis and tinea corporis, ecthyma has no epidermal hyperplasia.
(c) D. Psoriasis is characterized by an increased epidermal cell turnover, increased numbers of epidermal stem cells,
and an abnormal differentiation of keratin. This leads to the classic scale associated with psoriasis.
Ref: (10)

11
12. History & Physical/Pulmonology
On physical examination you note diminished breath sounds over the right lower lobe with decreased tactile fremitus
and dullness to percussion. Which of the following is the most likely cause?
A. Asthma
B. Pneumonia
C. Pneumothorax
D. Pleural effusion
Explanations
(u) A. Asthma is characterized by decreased tactile fremitus, but would have resonant to hyperresonant percussion,
not dullness.
(u) B. Lobar pneumonia is characterized by dullness to percussion, but would have an increased, not decreased,
tactile fremitus.
(u) C. A pneumothorax is characterized by decreased to absent tactile fremitus, but would have a hyperresonant
percussion note, not dullness.
(c) D. Decreased tactile fremitus and dullness to percussion would be found in a pleural effusion.
Ref: (1)

13. Clinical Therapeutics/Psychiatry/Behavioral Medicine


Which of the following is the treatment of choice for benzodiazepine intoxication?
A. Flumazenil (Romazicon)
B. Naloxone (Narcan)
C. Chlordiazepoxide (Librium)
D. Clonidine (Catapres)
Explanations
(c) A. Flumazenil is the treatment of choice for benzodiazepine intoxication.
(u) B. Naloxone is used for the treatment of opioid intoxication and withdrawal.
(u) C. Chlordiazepoxide is used for alcohol detoxification.
(u) D. Clonidine is an alpha blocker that is often used for withdrawal from opioids, alcohol, benzodiazepines, or
nicotine.
Ref: (14)

14. Diagnosis/Cardiology
A 56 year-old male with a known history of polycythemia suddenly complains of pain and paresthesia in the left leg.
Physical examination reveals the left leg to be cool to the touch and the toes to be cyanotic. The popliteal pulse is
absent by palpation and Doppler. The femoral pulse is absent by palpation but weak with Doppler. The right leg and
upper extremities have 2+/4+ pulses throughout. Given these findings what is the most likely diagnosis?
A. Venous thrombosis
B. Arterial thrombosis
C. Thromboangiitis obliterans
D. Thrombophlebitis
Explanations
(u) A. See B for explanation.
(c) B. Arterial thrombosis has occurred and is evidenced by the loss of the popliteal and dorsalis pedis pulse. This is
a surgical emergency. Venous occlusion and thrombophlebitis do not result in loss of arterial pulse.
(u) C. See B for explanation.
(u) D. See B for explanation.
Ref: (28)

12
15. Clinical Intervention/Obstetrics/Gynecology
An 18 year-old G1P0 female presents for her 35 week prenatal visit with complaints of headache, blurred vision and
right upper quadrant discomfort. Vital signs show BP of 170/100 mmHg and brisk patellar reflexes. Urinalysis shows
3+ proteinuria. Fetal heart tones are 150. What is your next step in the care of this patient?
A. Admit to hospital and prepare for delivery
B. Admit to hospital with antepartum fetal surveillance and close monitoring of maternal conditions
C. Order bed rest at home with daily fetal movement counts and twice weekly antepartum care
D. Order bed rest at home with administration of prophylactic magnesium sulfate
Explanations
(c) A. Severe preeclampsia mandates hospitalization. Delivery is indicated if gestational age is 34 weeks or
greater.
(u) B. Antepartum fetal surveillance and close monitoring in hospital is appropriate for pregnant female with
unfavorable cervix and mild preeclampsia.
(u) C. See A for explanation.
(u) D. See A for explanation.
Ref: (8)

16. Health Maintenance/Orthopedics/Rheumatology


Which of the following risk factors is the most predictive for the development of osteoarthritis?
A. Age
B. Major joint trauma
C. Prior inflammatory joint disease
D. Repetitive stress
Explanations
(c) A. Age is the most significant risk factor for osteoarthritis. Prevalence and severity increase with age. Ninety
percent of people greater than the age of 40 have degenerative changes of the weight bearing joints. Major
joint trauma, prior inflammatory joint disease, and repetitive stress are additional risk factors but not as
predictive as age.
(u) B. See A for explanation.
(u) C. See A for explanation.
(u) D. See A for explanation.
Ref: (28)

17. Diagnostic Studies/Urology/Renal


A 48 year-old female presents to the clinic complaining of hematuria. The patient states that she was found to have
hematuria during an insurance physical examination. The patient denies dysuria or frequency. She also denies pain
in the abdomen, flank or meatus. She denies any history of previous nephrolithiasis. Urinalysis reveals the urine to be
yellow and slightly hazy with a positive dipstick for hemoglobin. Microscopic reveals 5-7 RBCs/HPF without WBCs,
bacteria, casts, or crystals. What is the next diagnostic study this patient should undergo?
A. CT urography
B. Intravenous pyelogram
C. Abdominal ultrasound
D. Cystoscopy
Explanations
(c) A. CT urography with and without contrast should be done to evaluate the upper and lower urinary tract for
neoplasms, and benign conditions such as urolithiasis. This has replaced IVP for imaging of the upper tracts.
Abdominal ultrasound will not help in this scenario and the role of renal ultrasound in evaluation of hematuria is
unclear. Cystoscopy will help to assess the bladder and urethra but will not help with evaluation of the upper
urinary tract.

(u) B. See A for explanation.


(u) C. See A for explanation.
(u) D. See A for explanation.
Ref: (28)

13
18. Scientific Concepts/Infectious Diseases
The pathologic process responsible for the renal damage in post-streptococcal glomerulonephritis is which of the
following?
A. Immunologic
B. Vascular
C. Hormonal
D. Genetic
Explanations
(c) A. The antigen-antibody complex that occurs as a result of streptococcal infection is the result of an abnormal
immunologic response.
(u) B. See A for explanation.
(u) C. See A for explanation.
(u) D. See A for explanation.
Ref: (16)

19. History & Physical/Cardiology


A 25 year-old female presents with a three day history of chest pain aggravated by coughing and relieved by sitting
and leaning forward. She is febrile and a CBC with differential reveals leukocytosis. Which of the following physical
examination signs is characteristic of her problem?
A. Pulsus paradoxus
B. Localized crackles
C. Pericardial friction rub
D. Wheezing
Explanations
(u) A. Pulsus paradoxus is a classic finding for cardiac tamponade.
(u) B. Localized crackles are associated with pneumonia and consolidation, not pericarditis.
(c) C. Pericardial friction rub is characteristic of an inflammatory pericarditis.
(u) D. Wheezing is characteristic for pulmonary disorders, such as asthma.
Ref: (28)

20. Clinical Therapeutics/Gastrointestinal/Nutritional


Which of the following medications is most useful in maintaining remission in a patient with ulcerative colitis?
A. Oral corticosteroids
B. Corticosteroid enemas
C. Sulfasalazine
D. Macrodantin
Explanations
(u) A. Corticosteroids, given orally or via enema preparations, are used for acute flare-up of ulcerative colitis but are
not effective when given as prophylactic agents.
(u) B. See A for explanation.
(c) C. Sulfasalazine, olsalazine, and mesalamine are effective in maintaining remission in patients with ulcerative
colitis.
(u) D. Long-term antibiotic therapy with Macrodantin is used as a prophylactic agent to prevent urinary tract infections
and has no role in the treatment or prevention of ulcerative colitis.
Ref: (7)

14
21. Diagnosis/Endocrinology
An adult presents with a three month history of progressive severe muscle cramps, extremity paresthesias and
lethargy which began shortly after a thyroidectomy for a malignant thyroid lesion. Which of the following is the most
likely diagnosis?
A. Hypoparathyroidism
B. Hypothyroidism
C. Hyperparathyroidism
D. Hyperthyroidism
Explanations
(c) A. Hypocalcemia secondary to hypoparathyroidism is commonly seen as a complication of thyroidectomy.
(u) B. Hypothyroidism is possible without replacement therapy after thyroidectomy but would not result in the tetany-
like symptoms and hypocalcemia.
(u) C. Hyperparathyroidism will have a different constellation of symptoms and will be typified by elevated serum
calcium.
(u) D. Hyperthyroidism is not likely status post thyroidectomy though possible with over-aggressive replacement
therapy. Symptoms with low serum calcium are the factors that move away from this being the diagnosis.
Ref: (28)

22. Clinical Intervention/Pulmonology


A 22 year-old patient complains of sudden onset of chest pain accompanied by shortness of breath. The patient
appears dyspneic. On examination, the trachea is deviated to the left, breath sounds are faint on the right, and the
right chest is hyperresonant to percussion. The preferable treatment for this patient would be
A. a tracheostomy.
B. insertion of a chest tube with underwater seal, left 2nd intercostal space.
C. needle thoracotomy right 2nd intercostal space
D. a lung scan for pulmonary embolus and begin heparin sodium (Heparin) therapy.
Explanations
(u) A. Tracheostomy is indicated for upper airway obstruction, not spontaneous pneumothorax.
(h) B. This patient has a tension pneumothorax on the right, and insertion of the chest tube on the left would be life
threatening.
(c) C. Decreased breath sounds and hyperresonance are noted on the side of the pneumothorax; tracheal deviation
to the opposite side indicates development of a tension pneumothorax. Treatment consists of inserting a chest
tube on the side of the pneumothorax and connecting to an underwater seal.
(u) D. Pulmonary embolus may present with chest pain and dyspnea, but physical examination findings do not
include decreased breath sounds with hyperresonance and tracheal deviation.
Ref: (28)

23. Diagnostic Studies/ENT/Ophthalmology


A patient presents with acute eye pain and photophobia after putting in his contact lens. The patient states he
removed his contact which he noted to be torn. Which of the following is the diagnostic study of choice in this patient?
A. Funduscopic examination
B. Applanation tonometry
C. Fluorescein staining
D. Schirmer Test
Explanations
(u) A. Funduscopic examination allows for evaluation of the optic disc and retinal circulation. It is not the best
diagnostic study for a patient with suspected corneal abrasion.
(u) B. Applanation tonometry is used in the evaluation of intraocular pressure in patients with suspected glaucoma.
(c) C. Fluorescein staining is done in patients to evaluate for corneal abrasion which commonly presents with
severe eye pain and photophobia.
(u) D. The Schirmer test is used to evaluate the quantity of tears produced in patient with conditions such as
keratoconjunctivitis sicca.
Ref: (29)

15
24. History & Physical/Psychiatry/Behavioral Medicine
A 24 year-old female presents to your office for a physical examination. She is dressed in a low cut blouse and a
short skirt. She is dramatic, emotional and sexually provocative. She complains of difficulty being intimate with men.
On further questioning, she seems to overemphasize the severity of her current cold. After a full history and physical
examination you suspect what personality disorder?
A. Histrionic
B. Borderline
C. Narcissistic
D. Antisocial
Explanations
(c) A. Patients who are histrionic are attention seekers, and exaggerate their thoughts and feelings, they are
often sexually provocative.
(u) B. Borderline patients have unstable behavior and mood. They often have feelings of aloneness and self-
destructive behavior.
(u) C. Narcissistic patients are grandiose, envious, and have a sense of special entitlement. They lack empathy.
(u) D. Antisocial patients are unwilling to conform to social norms and do not learn from prior experiences.
Ref: (14)

25. Diagnosis/Cardiology
During physical examination an elderly patient is noted to have a painless, brown-colored ulceration in the area of the
medial malleolus. Which of the following is the most likely diagnosis?
A. Arterial ulcer
B. Venous ulcer
C. Arterial insufficiency
D. Diabetic ulcer
Explanations
(u) A. Arterial ulcers typically are the last in the sequence of events of peripheral arterial disease; which include
decreased or absent pulses distal to the blockage, muscle atrophy, hair loss, thickened nails, smooth and
shiny skin, reduced skin temperature, pallor, cyanosis, ulcers, and gangrene. Arterial ulcers typically occur on
the feet in the areas of pressure points.
(c) B. Venous ulceration develops in the lower extremity secondary to venous incompetence and chronic edema. The
medial aspect of the ankle is the most common location.
(u) C. Arterial insufficiency is most likely to present with symptoms of claudication prior to the development of skin
ulcers. When these ulcers do occur, they are most commonly seen as arterial ulcers and not venous ulcers.
(u) D. Diabetic ulcers typically occur due to atherosclerosis and arterial insufficiency along with diabetic peripheral
neuropathy. These ulcers are more likely to occur on pressure points on the foot and fail to heal because of
poor circulation to these areas.
Ref: (7)

26. Health Maintenance/Obstetrics/Gynecology


A 30 year-old female presents to the clinic for her 6 week post partum examination. She was diagnosed with
gestational diabetes mellitus during her pregnancy and was successfully treated with diet alone. Which of the
following patient education statements is most appropriate for this patient?
A. The risk of gestational diabetes occurring in future pregnancies is very low.
B. The risk of developing diabetes in 10-15 years is relatively low.
C. Lifestyle modification, diet & exercise will not prevent her from developing diabetes
D. Glucose testing every 1-3 years is recommended
Explanations
(u) A. The risk of recurrence in future pregnancies is 60%.
(u) B. She has a 50-60% risk of developing diabetes within 10-15 years.
(u) C. Lifestyle modification may delay or prevent developing diabetes.
(c) D. Those with normal glucose should be reassessed every 3 years; those with prediabetes should be
assessed annually.
Ref: (8)

16
27. Clinical Therapeutics/Neurology
A patient with known relapsing-remitting multiple sclerosis (MS) presents to the hospital with an exacerbation of MS
symptoms. What is the recommended treatment?
A. Baclofen (Lioresal)
B. Interferon
C. Glatiramer acetate (Copaxone)
D. Methylprednisolone
Explanations
(u) A. See D for explanation.
(u) B. See D for explanation.
(u) C. See D for explanation.
(c) D. Acute relapses of MS are treated with a short course of IV methylprednisolone followed by oral prednisone.
This regimen reduces the severity and shortens the duration of attacks. All other drugs listed are used to
reduce the attack rate of relapsing remitting multiple sclerosis.
Ref: (1)

28. Clinical Intervention/Dermatology


A 3 year-old child playing in an abandoned shed is bitten by a black widow spider. The mother rushes the child to the
emergency department within 20 minutes of the incident. Which of the following if the best initial intervention?
A. Intramuscular steroids
B. Administration of antivenin
C. Immediate immersion in a cold bath
D. Hospital admission for symptomatic care
Explanations
(u) A. The bite of this spider does not cause an inflammatory reaction; therefore, steroids are not indicated.
(u) B. Administration of antivenin should be administered to patients with severe envenomation manifested as
seizures, respiratory failure, or hypertension.
(u) C. Application of an ice pack initially would be helpful for pain relief, however, immersion in a cold bath may
cause hypothermia.
(c) D. Hospital admission for symptomatic care should be considered in children, pregnant women, and patients
with preexisting cardiovascular disease.
Ref: (17)

29. Diagnostic Studies/Orthopedics/Rheumatology


The most accurate way to determine the exact degree of spinal curvature in a child with scoliosis is by which of the
following?
A. Calculation of the Cobb angle
B. Measurement of waist asymmetry
C. Measurement of rib hump deformity
D. Calculation using a scoliometer
Explanations
(c) A. The scoliotic curve is measured by the Cobb method using AP and lateral x-ray films of the entire length of the
spine.
(u) B. While waist asymmetry and rib hump deformity may be observed on physical examination of the patient with
scoliosis, none of them can be used to determine the exact degree of the spinal curvature.
(u) C. See B for explanation.
(u) D. A scoliometer or inclinometer measures distortions of the torso and is good for screening angle of rotation, but
is not exact to determine exact degree of curvature.
Ref: (26)

17
30. Scientific Concepts/Urology/Renal
Which of the following is the portion of the nephron responsible for the absorption of 90% of the ultrafiltrate?
A. Proximal convoluted tubule
B. Loop of Henle
C. Distal convoluted tubule
D. Collecting duct
Explanations
(c) A. The majority of the ultrafiltrate 90% is reabsorbed in the proximal convoluted tubule.
(u) B. The loop of Henle is responsible for the concentration of solutes within the nephron.
(u) C. The distal convoluted tubule is responsible for some water and sodium reabsorption.
(u) D. The collecting duct is responsible for the final concentration of the urine.
Ref: (1)

31. Health Maintenance/Cardiology


Which of the following is a proven risk factor for the development of abdominal aortic aneurysm?
A. Infective endocarditis
B. Diabetes mellitus
C. Cigarette smoking
D. Alcohol abuse
Explanations
(u) A. Infective endocarditis is not associated with the development of abdominal aortic aneurysm.
(u) B. Diabetic patients do have a higher rate of atherosclerosis, but there is no clear causal evidence of diabetics
being at higher risk for the development of abdominal aortic aneurysm.
(c) C. Cigarette smoking is the primary risk factor for the development of aortic aneurysms.
(u) D. Alcohol abuse is not related to development of abdominal aortic aneurysm.
Ref: (30)

32. Clinical Therapeutics/Pulmonology


A 22 month-old male infant presents with one day of barking cough preceded by three days of cold symptoms. On
physical examination, his axillary temperature is 100.4F and he has no stridor at rest. Inspiratory stridor is evident
when he becomes agitated during the examination. There are no signs of respiratory distress or cyanosis. Which of
the following is the most appropriate treatment for this patient?
A. Nebulized albuterol
B. Nebulized epinephrine
C. Oral amoxicillin
D. Oral dexamethasone
Explanations
(u) A. Viral croup is an upper airway disease and there is no role for bronchodilator therapy.
(u) B. Nebulized racemic epinephrine is only indicated in the treatment of croup in cases of moderate to severe
rest stridor, respiratory distress, or hypoxia.
(u) C. Croup is almost always a viral illness and antibiotics have no role in the treatment of this condition.
(c) D. Corticosteroids are beneficial in the treatment of croup. Intramuscular administration has shown no benefit
over oral administration.
Ref: (5)

18
33. History & Physical/ENT/Ophthalmology
A patient with history of hypertension and dyslipidemia presents for routine follow up. On funduscopic examination
you note moderate sized fluffy white lesions with irregular borders. This is most consistent with which of the
following?
A. Drusen
B. Cotton-wool patches
C. Hard exudates
D. Preretinal hemorrhages
Explanations
(u) A. Drusen are tiny to small yellowish round spots with hard or soft edges that are often seen in age-related
macular degeneration.
(c) B. Cotton-wool patches are fluffy white or grayish ovoid lesions with irregular borders. They are typically
moderate in size and seen in patients with hypertension.
(u) C. Hard exudates are yellowish bright lesions with well-defined borders. They are often small and round.
(u) D. Preretinal hemorrhages obscure the underlying retinal vessels and are seen as a horizontal line of
demarcation with plasma above and cells below.
Ref: (3)

34. Diagnosis/Gastrointestinal/Nutritional
A 50 year-old male with history of alcohol abuse presents with acute, severe epigastric pain radiating to the back. The
patient admits to an episode of coffee ground emesis. On examination he is ill-appearing with a rigid, quiet abdomen
and rebound tenderness. Which of the following is the most likely diagnosis?
A. Abdominal aortic aneurysm
B. Perforated duodenal ulcer
C. Acute myocardial infarction
D. Cholecystitis
Explanations
(u) A. A patient with an abdominal aortic aneurysm may present with pain radiating to the back, however would not
have coffee ground emesis or an acute abdomen.
(c) B. Perforation of a duodenal ulcer causes sudden, severe pain, with rebound tenderness and rigid abdomen on
physical examination. It is often associated with coffee ground emesis.
(u) C. A patient with an acute myocardial infarction may have pain radiating to the back, however would not have
hematemesis or an acute abdomen.
(u) D. Cholecystitis presents with right upper quadrant pain and is not typically associated with coffee ground
emesis or rebound tenderness.
Ref: (1)

35. Clinical Intervention/Endocrinology


A 58 year-old female presents with acute onset of 105 degrees F fever, chills, delirium and tachycardia. Laboratory
analysis reveals a TSH of 0.08 mcU/L (0.4-6.0 mcU/L), total T3 of 400 ng/dL (95-190 ng/dL)and a total T4 of 180
mcg/dL (5-11 mcg/dL). What is the initial treatment of choice to normalize this condition?
A. Hydrocortisone
B. Propylthiouracil
C. Digoxin
D. Levothyroxine
Explanations
(u) A. The use of steroids is common in thyroid storm, but it is not the best initial treatment.
(c) B. Propylthiouracil is the preferred initial drug in patients with thyroid storm.
(u) C. Digoxin is not the treatment of choice for symptoms related to thyroid storm. It could be used if heart failure
was associated.
(h) D. Levothyroxine would actually worsen this condition.
Ref: (29)

19
36. Clinical Therapeutics/Cardiology
When instituting diuretic therapy for patients with heart failure, which of the following is considered the treatment of
choice as first-line therapy in a failing kidney due to its improved sodium clearance?
A. Hydrochlorothiazide (Diuril)
B. Bumetanide (Bumex)
C. Spironolactone (Aldactone)
D. Acetazolamide (Diamox)
Explanations
(u) A. Thiazide diuretics may have better hypertension control than the short acting loop diuretics but they are
generally ineffective when the glomerular filtration rate falls below 30-40 mL/min.
(c) B. Loop diuretics remain active in severe renal insufficiency and are the most effective type of diuretics used in
the management of heart failure symptoms. These agents have a rapid onset and result in natriuresis due to
their activity in the ascending limb in the Loop of Henle.
(u) C. Potassium-sparing diuretics have very weak diuretic properties and are useful as adjunctive therapy in
patients with Stage 3 or Stage 4 heart failure who are already on a first-line diuretic and other agents.
(u) D. Acetazolamide is a carbonic anhydrase inhibitor and works by causing a metabolic acidosis with loss of
bicarbonate in the failing kidney. It is a weak diuretic and does not work in the setting of a failing kidney.
Ref: (28)

37. History & Physical/Hematology


A female patient presents with weakness and fatigue for the past three months. A CBC was ordered and revealed the
following values:

RBC 3.8 X 10(6)mcL (4.7-6.1 x 10(6)mcL)


HCT 27% (35-45%)
Hgb 9.2 gms/dL (12-15.5 gms/dL)
MCV 120 fL (80-100 fL)
MCH 35 pg (26-34 pg)

On physical examination which of the following would be a consistent finding for this type of anemia?
A. Paresthesia of the hands and feet
B. Hepatosplenomegaly
C. Tachycardia
D. Jaundice
Explanations
(c) A. Neurological manifestation is the earliest type most commonly seen with megaloblastic anemia most
commonly from vitamin B12 deficiency.
(u) B. Hepatosplenomegaly and jaundice are commonly seen in hemolytic anemias.
(u) C. This compensatory mechanism is common in any type of anemia due to the hypoxemic effects of the anemia.
(u) D. See B for explanation.
Ref: (28)

38. Scientific Concepts/Pulmonology


A 20 year-old male presents with 3 weeks of constitutional and upper respiratory symptoms, including malaise, sore
throat, dry cough, and fever. Lung auscultation demonstrates diffuse crackles bilaterally. What is the most likely
infectious agent involved?
A. Respiratory syncytial virus
B. Influenza virus
C. Mycoplasma pneumoniae
D. Streptococcus pneumoniae
Explanations
(u) A. RSV is characterized by wheezing on auscultation and the course is typically 3-7 days.
(u) B. Influenza pneumonia is characterized by a more precipitous onset and fulminant course.

20
(c) C. The indolent course suggests an atypical pneumonia and Mycoplasma is the most common atypical agent.
(u) D. Pneumococcal pneumonia is typically characterized by a more severe illness and more fulminant course.
Ref: (7)

39. Diagnostic Studies/Obstetrics/Gynecology


A 30 year-old female presents to the emergency room having passed out at home 30 minutes prior to arrival. Her last
menstrual period was 6 weeks ago and pregnancy test was reported positive 5 days ago. She started having vaginal
bleeding last night. Vital signs are BP 70/40 mmHg, P 140 bpm, R 22. She is pale and diaphoretic. The next step in
the evaluation of this patient's condition should be which of the following?
A. Abdominal ultrasound
B. Culdocentesis
C. Laparotomy
D. Magnetic resonance imaging
Explanations
(u) A. Abdominal ultrasound is not the test of choice in this patient.
(u) B. Culdocentesis has been replaced by transvaginal ultrasound as diagnostic procedure for suspected ectopic
pregnancy.
(c) C. Laparotomy is indicated with presumptive diagnosis of ectopic pregnancy in an unstable patient.
(u) D. MRI is useful in cases of unusual ectopic locations in stable patients.
Ref: (8)

40. Diagnosis/ENT/Ophthalmology
A 35 year-old patient presents with a sudden onset of fever, dysphonia, drooling, and difficulty drinking a few hours
ago. Physical examination reveals a temperature of 102 degrees F. The patient appears ill and is sitting forward.
Inspiratory retractions are noted and there is a soft stridor. Which of the following is the most likely diagnosis?
A. Angioedema
B. Foreign body aspiration
C. Epiglottitis
D. Bacterial pharyngitis
Explanations
(u) A. Angioedema would present with swelling of the mouth and upper airway. Patient would not have fever or
inspiratory retractions.
(u) B. Patients with foreign body aspiration are unlikely to appear acutely ill or be febrile.
(c) C. Epiglottitis is characterized by fever, dysphonia, drooling, and dysphagia. Patients often appear ill and sit up
leaning forward in an attempt to breathe.
(u) D. Bacterial pharyngitis is not associated with stridor or inspiratory retractions.
Ref: (28)

41. Clinical Intervention/Orthopedics/Rheumatology


A 53 year-old female massage therapist presents with new onset of sudden swelling involving the right elbow. She
denies previous episodes similar to this. On physical examination, the patient is afebrile. There is a 4 cm fluid-filled
mass that is tender to palpation overlying the tip of the elbow with no evidence of erythema or warmth. Which of the
following is the most appropriate intervention?
A. Rest and NSAIDs
B. Surgical excision
C. Incision and drainage
D. Aspiration and corticosteroid injection
Explanations
(c) A. Rest and NSAID is the most appropriate initial intervention in a patient with olecranon bursitis.
(u) B. Surgical excision is reserved for chronic bursitis and is not indicated in this acute initial presentation.

21
(u) C. Incision and drainage is not recommended as it may cause a chronic drainage sinus tract.
(u) D. Aspiration of the bursa and corticosteroid injection are second-line therapy in a patient with olecranon
bursitis who fails rest and NSAIDs.
Ref: (28)

42. History & Physical/Gastrointestinal/Nutritional


A patient presents with nausea, vomiting, and fever for one day. She describes periumbilical abdominal pain which
has settled in her right lower quadrant. On examination she has right lower quadrant pain with deep left lower
quadrant palpation. This finding is known as which of the following?
A. Psoas sign
B. Rebound tenderness
C. Rovsing's sign
D. Obturator sign
Explanations
(u) A. Abdominal pain with flexion of the right hip against pressure is a psoas sign.
(u) B. Right lower quadrant pain with quick withdrawal of the right lower quadrant palpation is rebound
tenderness.
(c) C. Right lower quadrant pain with deep left lower quadrant palpation is Rovsing's sign.
(u) D. Right lower quadrant pain with internal rotation of the right hip while both the hip and knee are flexed is
obturator sign.
Ref: (3)

43. Clinical Therapeutics/Urology/Renal


A 66 year-old male patient has both hypertension and benign prostatic hypertrophy. Which of the following
medications would most likely benefit both conditions?
A. Finasteride (Propecia)
B. Verapamil (Calan)
C. Propanolol (Inderal)
D. Prazosin (Minipress)
Explanations
(u) A. This drug could help to relieve the symptoms of benign prostatic hypertrophy (BPH) however it has no
benefit for hypertension.
(u) B. This drug could help the HTN however it would have no benefit of helping the BPH.
(u) C. See B for explanation.
(c) D. This drug has antihypertensive properties as well as properties to relieve the symptoms of BPH.
Ref: (28)

44. Diagnostic Studies/Cardiology


High ventricular filling pressures are indicated by a rise in which of the following?
A. Brain natriuretic peptide
B. Troponin
C. Myoglobin
D. Creatinine
Explanations
(c) A. Brain natriuretic peptide (BNP) is a hormone released from the myocardium when stretched such as with high
ventricular filling pressures from CHF. It can indicate or be used as prognostic evaluation in patients with
acute CHF.
(u) B. Troponin is a marker for cardiac muscle damage as in an acute myocardial infarction or ischemia.
(u) C. Myoglobin is a byproduct of muscle destruction or damage from ischemia or other causes of skeletal muscle
damage. Myoglobin may rise with high ventricular pressures, however it is nonspecific.
(u) D. Creatinine is a marker of renal function.
Ref: (28)

22
45. Health Maintenance/Infectious Diseases
Which of the following prophylactic interventions is recommended in all individuals with HIV regardless of disease
stage?
A. Oral polio vaccine
B. Pneumococcal vaccine
C. Fluconazole for fungal infections
D. Isoniazid for tuberculosis reactivation
Explanations
(u) A. Oral polio vaccine is contraindicated in HIV patients because it is a live attenuated vaccine.
(c) B. All HIV-positive individuals should receive prophylaxis against pneumococcal pneumonia.
(u) C. Prophylaxis against fungal infection would be indicated for those HIV-positive individuals with CD4 less than
50 cells/L.
(u) D. Isoniazid is indicated for those with positive PPD and normal chest x-ray. Reactivation would require a
multidrug regimen.
Ref: (1)

46. Diagnosis/Dermatology
A 60 year-old male presents with a slowly developing facial lesion first noticed 4-5 months ago. He describes it as
non-painful and non-pruritic but notes it to be extremely scaly. He denies a history of similar lesions or dermatologic
disease. Examination reveals a one centimeter, firm nodule at the right temple with heavy keratinization. There is no
fluctuance or skin discoloration. Which of the following is the most likely diagnosis?
A. Actinic keratosis
B. Squamous cell cancer
C. Granuloma annulare
D. Merkel cell carcinoma
Explanations
(u) A. Actinic keratosis lesions can be very scaly but are generally flat in appearance not nodular.
(c) B. This is a very typical scenario for a squamous cell cancer occurring in a sun exposed area, with slow
development and heavy keratinization.
(u) C. Granuloma annulare is a self-limited dermatosis occurring primarily on the distal extremities is more
common in young adults and children.
(u) D. Merkel cell carcinoma lesions are typically discolored and non-keratinizing.
Ref: (10)

47. Clinical Intervention/Psychiatry/Behavioral Medicine


A 25 year-old male on a behavioral medicine unit is given haloperidol (Haldol) IM for a violent psychotic outburst.
Initially he quiets down, but about an hour later develops confusion, an inability to open his mouth, and a temperature
of 40 degrees C. Initial treatment should consist of which of the following?
A. Additional Haldol
B. Corticosteroid
C. Benzodiazepine
D. Dantrolene
Explanations
(h) A. This patient is exhibiting symptoms of neuroleptic malignant syndrome (NMS) and an additional dose of a
antipsychotic will be harmful, and potentially deadly with a 20% mortality if untreated.
(u) B. Steroids will not reduce the symptoms of NMS.
(h) C. Benzodiazepines will not reduce the symptoms of NMS.
(c) D. In addition to supportive treatment, the most commonly used medications for neuroleptic malignant syndrome
are dantrolene (Dantrium) and bromocriptine (Parlodel).
Ref: (14)

23
48. History & Physical/Endocrinology
Which of the following is the most characteristic physical examination finding with Grave's disease?
A. Diffuse thyroid gland enlargement
B. Single thyroid gland nodule
C. Unilateral thyroid lobe enlargement
D. Multiple thyroid gland nodules
Explanations
(c) A. Diffuse thyroid gland enlargement is the typical presentation for Grave's disease.
(u) B. Single thyroid gland nodule would be more worrisome for thyroid malignancy.
(u) C. Unilateral thyroid lobe enlargement is an atypical presentation for any thyroid malady.
(u) D. Multiple thyroid gland nodules are more suggestive of a metabolic disorder.
Ref: (3)

49. Diagnostic Studies/Pulmonology


Which of the following chest x-ray abnormalities would most likely be seen in a patient with hypersensitivity
pneumonitis?
A. Lobar consolidation
B. Apical infiltration
C. Granulomatous inflammation
D. Diffuse nodular densities
Explanations
(u) A. Lobar consolidation is seen in community-acquired pneumonia.
(u) B. Apical infiltration is seen in tuberculosis.
(u) C. Granulomatous inflammation is seen in sarcoidosis.
(c) D. Diffuse nodular densities are seen in hypersensitivity pneumonitis.
Ref: (7)

50. Clinical Therapeutics/Obstetrics/Gynecology


A 32 year-old female complains of severe irritability and emotional lability accompanied by weight gain, breast
tenderness, and headache starting mid-cycle each month and abating with the onset of menses. She has tried
relaxation therapy, exercise, dietary changes and over the counter pharmacologic interventions with minimal relief of
her symptoms. Which of the following prescription medications is the best choice to relieve her symptoms?
A. Alprazolam (Xanax)
B. Fluoxetine (Prozac)
C. Oral contraceptive pill (Ortho-novum 1/35)
D. Spironolactone (Aldactone)
Explanations
(u) A. Anxiolytics have shown to be effective however its potential for dependency makes it not the best choice.
(c) B. SSRI's provide symptom improvement for patients with premenstrual syndrome (PMS).
(u) C. Studies have found little difference between women taking a low dose birth control and women who do not
take pills; currently not recommended for PMS.
(u) D. Spironolactone is used for cyclic edema, not the best choice for PMS.
Ref: (8)

24
51. Diagnosis/Orthopedics/Rheumatology
A 30 year-old male sustains a blow to his right lateral leg during a soccer game. He complains of pain with weight
bearing. Examination reveals tenderness along the lateral aspect of the right lower leg, but no point tenderness over
the tibia. There is full active range of motion at the ankle, knee, and hip joints. There is no swelling or tenderness of
the ankle or knee joints. Which of the following is the most likely diagnosis?
A. Anterior cruciate ligament tear
B. Fractured fibula
C. Tibial stress fracture
D. Gastrocnemius contusion
Explanations
(u) A. Anterior cruciate ligament injury would have a positive drawer test and mechanism of injury can be due to a
direct blow to the knee or as a result of sudden deceleration and rotation of the knee.
(c) B. Isolated fibular fractures can occur with direct or indirect trauma to the fibular shaft. X-ray films of the leg are
mandatory for any patient with a history of trauma and pain on ambulation to rule out this potentially
overlooked injury.
(u) C. A tibial stress fracture is more likely to present without acute injury and with tenderness over the tibia rather
than the lateral aspect of the leg.
(u) D. A contusion does not cause pain with weight bearing.
Ref: (29)

52. Health Maintenance/Neurology


What is the recommendation for primary prevention of stroke in a patient under sixty years of age with atrial
fibrillation?
A. No therapy is needed
B. Aspirin
C. Warfarin
D. Maze procedure
Explanations
(c) A. No therapy is recommended for primary stroke prevention in this patient.
(u) B. See A for explanation.
(u) C. See A for explanation.
(u) D. See A for explanation.
Ref: (1)

53. Clinical Intervention/Cardiology


Which of the following is first-line treatment for a symptomatic bradyarrhythmia due to sick sinus syndrome?
A. Permanent pacemaker
B. Radiofrequency ablation
C. Antiarrhythmic therapy
D. Anticoagulation therapy
Explanations
(c) A. Permanent pacemakers are the therapy of choice in patients with symptomatic bradyarrhythmias in sick sinus
syndrome.
(u) B. Radiofrequency ablation is used for the treatment of accessory pathways in the heart.
(u) C. See A for explanation.
(u) D. See A for explanation.
Ref: (28)

25
54. Scientific Concepts/ENT/Ophthalmology
Which of the following is the most common cause of conductive hearing loss in an adult patient?
A. Head trauma
B. Cerumen impaction
C. Otosclerosis
D. Diabetes mellitus
Explanations
(u) A. Head trauma and diabetes mellitus are causes of sensorineural hearing loss.
(c) B. Cerumen impaction is the most common cause of conductive hearing loss in an adult patient.
(u) C. Although a cause of conductive hearing loss, otosclerosis is not the most common cause.
(u) D. See A for explanation.
Ref: (28)

55. History & Physical/Urology/Renal


A newborn male infant is seen by the PA prior to discharge home. On examination it is noted that the meatal opening
is on the dorsal surface of the penile shaft just behind the coronal sulcus. The testicles are descended bilaterally. This
physical examination finding is called which of the following?
A. Penile epispadias
B. Penile hypospadias
C. Glandular epispadias
D. Glandular hypospadias
Explanations
(c) A. This is a penile epispadias which can develop anywhere along the penile shaft to the pubic bone. It is caused
by displacement of urethra/meatal opening on the dorsal surface of the penis. If the opening occurs in the
glans penis then it would be a glandular epispadias. If this abnormality occurs on the ventral surface of the
penis or glans penis then it is known as a hypospadias.
(u) B. See A for explanation.
(u) C. See A for explanation.
(u) D. See A for explanation.
Ref: (33)

56. Diagnostic Studies/Gastrointestinal/Nutritional


Which of the following is the study of choice to diagnose upper gastrointestinal malignancy?
A. Abdominal CT
B. Upper endoscopy
C. Barium swallow
D. Abdominal ultrasound
Explanations
(u) A. See B for explanation.
(c) B. Upper endoscopy is the study of choice to diagnose gastroduodenal ulcers, erosive esophagitis and upper
gastrointestinal malignancy.
(u) C. See B for explanation.
(u) D. See B for explanation.
Ref: (28)

26
57. Clinical Therapeutics/Pulmonology
A 17 year-old girl uses an albuterol inhaler to treat her asthma. She uses the inhaler as needed and reports
symptoms occurring 3-4 days per week. She experiences symptoms at night no more than once a month. Her
spirometry during her most recent office visit is normal. What is the appropriate medical management of this patient?
A. Add an inhaled long-acting bronchodilator
B. Add an inhaled steroid
C. Add an inhaled long-acting bronchodilator and steroid
D. No change to her medical regimen
Explanations
(u) A. See B for explanation.
(c) B. Symptoms occurring more than twice a week but less than daily meet severity criteria for mild persistent
asthma. The appropriate next step in her therapy is to add an inhaled steroid. Adding an inhaled long-acting
bronchodilator is only recommended for moderate persistent asthma and only after an inhaled steroid has
been added.
(u) C. See B for explanation.
(u) D. See B for explanation.
Ref: (28)

58. Diagnosis/Psychiatry/Behavioral Medicine Topic: 1d Author: Kathleen Ehrhardt


A 24 year-old male presents to the emergency room via ambulance. He was found by the police walking naked on
the highway. His speech is minimal but disorganized and he appears to be responding to auditory hallucinations. With
further questioning, he is preoccupied with the delusion of the FBI listening in on his conversations. His mother tells
you that he has been acting bizarre for two months now. What is the most likely diagnosis?
A. Schizophrenia
B. Schizophreniform disorder
C. Schizoaffective disorder
D. Schizotypal personality disorder
Explanations
(u) A. Schizophrenia occurs with the above symptoms of greater than six months duration.
(c) B. Schizophreniform disorder is characterized by the same features as schizophrenia except the total duration
of the illness is at least one month and less than six months.
(u) C. Schizoaffective disorder has the features of both schizophrenia and a mood disorder.
(u) D. Patients with schizotypal personality disorder are strikingly odd or strange. They have magical thinking and
derealization.
Ref: (14)

59. History & Physical/Dermatology


What is the most common examination finding in a patient diagnosed with mumps?
A. Occipital lymph node swelling
B. Parotid gland tenderness
C. Splenic enlargement
D. Testicular tenderness
Explanations
(u) A. Lymph node swelling can occur secondary to facial edema but can be variable and more often involves the
submandibular and cervical lymph node chains.
(c) B. Parotid gland tenderness and swelling are the hallmark findings of mumps.
(u) C. Splenic enlargement is not a common exam finding even in extra-salivary disease.
(u) D. Testicular tenderness can occur in as many as 40% of post-pubescent men but it is not the most common
finding considering all cases of mumps.
Ref: (28)

27
60. Scientific Concepts/Cardiology
What is the most likely mechanism responsible for retinal hemorrhages and neurologic complications in a patient with
infective endocarditis?
A. Metabolic acidosis
B. Embolization of vegetations
C. Hypotension and tachycardia
D. Activation of the immune system
Explanations
(u) A. See B for explanation.
(c) B. The vegetations that occur during infective endocarditis can become emboli and can be dispersed throughout
the arterial system.
(u) C. See B for explanation.
(u) D. Glomerulonephritis and arthritis result from activation of the immune system.
Ref: (7)

61. Health Maintenance/Endocrinology


A patient with adrenal insufficiency is taking hydrocortisone 25 mg daily. What should the patient do with the
hydrocortisone dose when they develop a minor illness such as a cold?
A. Stop the hydrocortisone until the illness resolves.
B. Continue the current dose that the patient is taking.
C. Increase the dose to 50 mg daily until the illness resolves.
D. Increase the dose to 250 mg daily until the illness resolves
Explanations
(h) A. Stopping the hydrocortisone would cause adrenal crisis.
(u) B. See C for explanation.
(c) C. To better mimic the normal physiologic response the baseline dose should be doubled for the duration of the
illness. Doses should be increased 5-10 fold with major events such as surgery.
(u) D. See C for explanation.
Ref: (1)

62. Diagnosis/Neurology
A 30 year-old female presents to the office complaining of generalized weakness and reduced exercise tolerance that
improves with rest. On physical examination you note the presence of bilateral eyelid ptosis, proximal muscle
weakness and normal reflexes. What is the most likely diagnosis?
A. Lambert-Eaton syndrome
B. Organophosphate intoxication
C. Multiple sclerosis
D. Myasthenia gravis
Explanations
(u) A. Common symptoms of Lambert-Eaton syndrome are proximal muscle weakness of lower limbs, cranial nerve
findings, and depressed or absent reflexes. Patients commonly have a malignancy.
(u) B. Patients with organophosphate intoxication have seizures, excessive secretions, wheezing and diaphoresis.
(u) C. Patients with multiple sclerosis have multiple lesions in time and space.
(c) D. Common symptoms of myasthenia gravis are fatigable weakness, ptosis, diplopia, and proximal muscle
weakness. The disease is more common in women in the 2nd and 3rd decade and in men older than 60.
Ref: (1)

28
63. History & Physical/Obstetrics/Gynecology
During a routine prenatal visit, the fundal height is found to be at the umbilical level. The number of weeks gestation
is estimated to be
A. 10 to 12.
B. 16 to 18.
C. 20 to 22.
D. 26 to 28.
Explanations
(u) A. At 12 weeks, fundal height is palpable just above the pubic symphysis.
(u) B. At 16 weeks, fundal height is midway between the pubic symphysis and umbilicus.
(c) C. At 20 weeks, fundal height is at the umbilicus.
(u) D. At 26 weeks, fundal height is above the umbilicus.
Ref: (8)

64. Clinical Intervention/Gastrointestinal/Nutritional


An 18 month-old female presents to the Emergency Department having possibly swallowed a hearing aid battery
within the past hour. She is drooling and appears anxious but parents have noticed no stridor or dyspnea. She has no
history of previous esophageal injury. Physical examination is unremarkable. Chest radiograph reveals a radiopaque
round object at the distal esophagus. Which of the following is the most appropriate treatment option?
A. Observation for 24 hours
B. Esophagoscopy for removal
C. Barium swallow
D. Bronchoscopy
Explanations
(h) A. Batteries must be removed as they can induce mucosal injuries in as little as one hour of contact time.
(c) B. Esophagoscopy is the procedure of choice for acutely ingested foreign bodies.
(u) C. A barium swallow is a diagnostic option but will not provide treatment.
(u) D. Bronchoscopy would be the procedure of choice for an airway foreign body, not esophageal.
Ref: (29)

65. Diagnostic Studies/Cardiology


Which of the following is a non-invasive quick method of evaluating a patient with suspected lower extremity arterial
insufficiency?
A. Ankle-Brachial Index
B. Striker Tonometry
C. CT Angiography
D. Lower extremity arteriography
Explanations
(c) A. The single most useful index is the ankle pressure. This can be obtained with an Ankle-Brachial Index (ABI)
which the severity of signs and symptoms of arterial insufficiency are correlated to the findings on the ABI. It is
a non-invasive study that can be performed in an office setting.
(u) B. Striker Tonometry is used to evaluate compartment pressures.
(u) C. CT angiography is an invasive test involving radiation and contrast that cannot be done in an office setting.
(u) D. Lower extremity arteriography is an invasive test involving radiation and contrast that cannot be done in an
office setting.
Ref: (1)

29
66. Clinical Therapeutics/ENT/Ophthalmology
A patient presents complaining of left eye discharge and eyes that were matted shut this morning. The patient denies
changes in visual acuity, but states that he is afraid to put his contacts in. On physical examination you note
erythematous conjunctivae and mucopurulent discharge of the left eye. The cornea is clear. Which of the following
topical agents is the treatment of choice in this patient?
A. Aminoglycoside (Tobrex)
B. Olopatadine (Patanol)
C. Cycloplegic
D. Prednisolone acetate
Explanations
(c) A. Topical aminoglycoside or fluoroquinolones are indicated in contact lens wearers with conjunctivitis to cover
for Pseudomonas infection.
(u) B. Patanol is indicated in patients with allergic, not bacterial, conjunctivitis.
(u) C. Topical cycloplegic agents and corticosteroids are not indicated in the treatment of bacterial conjunctivitis.
(u) D. See C for explanation.
Ref: (29)

67. Diagnosis/Hematology
A 69 year-old female presents to the clinic complaining of fatigue. Physical examination reveals lymphadenopathy,
splenomegaly and pale conjunctiva. The remainder of the examination is unremarkable. CBC reveals a
normochromic normocytic anemia. White blood cell count is 45,000/mm3, with a differential of 77% mature
lymphocytes, 3% eosinophils, 18% segmented neutrophils and 1% basophils and monocytes. Platelets appear
adequate in number. What is the most likely diagnosis?
A. Acute myelocytic leukemia
B. Acute lymphocytic leukemia
C. Chronic lymphocytic leukemia
D. Chronic myelocytic leukemia
Explanations
(u) A. Acute leukemias present with circulating blast cells with pancytopenia.
(u) B. See A for explanation.
(c) C. Chronic lymphocytic leukemia typically occurs after age 50 presenting with lymphocytosis with lymphocytes
that appear small and mature.
(u) D. See C for explanation.
Ref: (28)

68. Health Maintenance/Pulmonology


A 62 year-old female is admitted to a nursing home during an outbreak of influenza. In review of her records, you
note that she did not receive the flu vaccine this year. Which of the following is the most appropriate drug of choice
for influenza prophylaxis in this patient?
A. Ciprofloxin (Cipro)
B. Oseltamivir phosphate (Tamiflu)
C. Clarithromycin (Biaxin)
D. Alpha-2b interferon (Avonex)
Explanations
(u) A. Ciprofloxin is indicated for postexposure prophylaxis of anthrax.
(c) B. Either oseltamivir or zanamivir are indicated for prophylactic use against influenza A or B.
(u) C. Clarithromycin is indicated for prophylaxis against disseminated Mycobacterium avium complex.
(u) D. Alpha-2b interferon is indicated for treatment of several disorders, such as chronic hepatitis B & C, but has no
role in prophylactic treatment of any condition.
Ref: (28)

30
69. Diagnostic Studies/Psychiatry/Behavioral Medicine
What laboratory test must be monitored frequently in patients who are taking clozapine (Clozaril)?
A. Thyroid stimulation hormone
B. White blood cell count
C. Platelet count
D. Aspartate aminotransferase
Explanations
(u) A. See B for explanation.
(c) B. Leukopenia, granulocytopenia, and agranulocytosis occur in approximately 1% of patients on this medication,
clozapine should not be dispensed without proof of monitoring.
(u) C. See B for explanation.
(u) D. See B for explanation.
Ref: (14)

70. Scientific Concepts/Orthopedics/Rheumatology


During the stages of fracture healing which of the following is responsible for producing collagen?
A. Osteoclasts
B. Chondrocytes
C. Glycosaminoglycans
D. Fibroblasts
Explanations
(u) A. Osteoclasts are responsible for removing necrotic bone.
(u) B. Chondrocytes make up the articular cartilage.
(u) C. Glycosaminoglycans help form the osteon or vascular canal.
(c) D. Fibroblasts produce collagen during the inflammation stage of healing.
Ref: (26)

71. History & Physical/Cardiology


Which of the following is the earliest symptom for patients with left ventricular failure?
A. Dependent edema
B. Dyspnea on exertion
C. Congestion
D. Chest pain
Explanations
(u) A. Right ventricular failure is manifested by dependent edema and congestion in the lungs.
(c) B. Patients with left ventricular heart failure may be comfortable at rest and may experience their first symptoms
with dyspnea with conversation or with mild exertion.
(u) C. Right ventricular failure is manifested by congestion in the lungs with cough being a possible manifestation of
this congestion.
(u) D. Patients who have on-going left ventricular failure may have underlying coronary heart disease. When the
heart failure is progressing, chest pain (manifested by angina) and myocardial infarction may be manifestations
of ongoing, progressive heart failure. Acute decline in heart failure may lead to acute myocardial infarction but
this tends to be a very late symptom.
Ref: (28)

31
72. Clinical Therapeutics/Endocrinology
34 year-old female status-post trans-sphenoidal resection of pituitary adenoma presents with worsening polydipsia of
10-12 liters daily and polyuria within four days of discharge. A urinalysis reveals a specific gravity of 1.004 (1.001-
1.035) and shows decreased urine osmolality but is otherwise normal. Labs reveal mild hypernatremia. What is the
treatment of choice for this patient?
A. Glyburide
B. Methylprednisolone
C. Desmopressin
D. Quinapril
Explanations
(u) A. Glyburide is not indicated because this patient has diabetes insipidus, not diabetes mellitus.
(h) B. Use of steroids in diabetes insipidus will actually worsen renal free water secretion.
(c) C. The main treatment for diabetes insipidus is Desmopressin.
(u) D. Quinapril is not indicated due to the lack of diabetes mellitus and overt renal failure/insufficiency.
Ref: (28)

73. Diagnosis/Urology/Renal
A 9 year-old boy who has had cold-like symptoms for the past few days is brought to the clinic by his mother who
states that her son had gross hematuria this morning. Prior to the cold-like symptoms the boy has been in excellent
health. He is up-to-date on all of his immunizations. The patient does not have any edema, hypertension or purpura.
Urinalysis reveals the urine to be cola-colored with a 2+ positive protein and 2+ hemoglobin. Microscopic analysis
reveals 50-100 RBCs/HPF, no WBCs, bacteria, casts or crystals. What is the most likely diagnosis?
A. Post streptococcal glomerulonephritis (PSGN)
B. IgA nephropathy
C. Minimal change disease (MCD)
D. Membranous nephropathy
Explanations
(u) A. PSGN usually presents 2-3 weeks after a streptococcal infection (pharyngeal or skin) and usually presents
with nephritic symptoms (edema, hypertension, cola-colored urine). This is due to trapping of the
streptococcal antigen within the glomerulus
(c) B. IgA nephropathy presents after an upper respiratory illness with deposition of IgA within the mesangium of the
glomerulus.
(u) C. MCD is the most common nephrotic presentation (edema, hypoproteinemia, hyperlipidemia, >3.5 gms. of
proteinuria in 24 hours) in children following an upper respiratory illness. This patient is not exhibiting any of
these signs at this time.
(u) D. Membranous nephropathy is the most common cause of adult nephrotic syndrome.
Ref: (28)

74. Clinical Intervention/Neurology


A 50 year-old male presents to the ER with a complaint of severe headaches. These headaches are unilateral and he
describes the headache pain as steady and non-throbbing. He also complains of nasal congestion and rhinorrhea. He
also mentions that alcohol often triggers these headaches. What do you recommend for this patient?
A. High flow oxygen
B. Massage
C. Ibuprofen
D. Propranolol
Explanations
(c) A. This a description of a cluster headache (migrainous neuralgia), these headaches respond to oxygen by
mask 7-10 L/min for 15 minutes.
(u) B. Massage may be effective for patients with tension headaches.
(u) C. Ibuprofen is helpful for the treatment of tension type headaches.
(u) D. Propranolol is a preventative medication used for migraine headaches.
Ref: (1)

32
75. Diagnostic Studies/Dermatology
An 8 year-old child is brought in by his mother with a two day history of spreading, non-pruritic red rash. The rash was
preceded by moderate fever, sore throat and rhinorrhea. Examination reveals a moderately ill appearing child with a
fine, macular-papular rash on an erythematous base spread diffusely over the trunk with some accentuation in the
skin folds. The face is flush with perioral pallor. There is palpable anterior cervical lymphadenopathy. Which of the
following is the most appropriate diagnostic study to establish the diagnosis?
A. Shave biopsy
B. Epstein-Barr virus Ig G
C. Complete blood count
D. Throat culture
Explanations
(u) A. Disorders such as erythema multiforme (EM) minor favors the extremities while the EM major may favor the
trunk and have associated oral mucosa involvement but generally has blistering lesions. Direct
immunofluorescence studies are negative in these disorders.
(u) B. Epstein-Barr virus Ig G would not be helpful in that it screens for prior exposure to this virus or illness such as
mononucleosis.
(u) C. Though a complete blood count is commonly ordered and may be helpful in stratifying illness in this case it
would not be diagnostic as to the etiology.
(c) D. Scarlatina rash is due to infection with group A strep. A throat culture would be the most appropriate
diagnostic study to establish the diagnosis in this patient.
Ref: (13)

76. History & Physical/Pulmonology


Upon auscultation of a patient's lungs, there are harsh, hollow breath sounds which have a long inspiratory
component in the region of the suprasternal notch. Throughout the periphery of the lung fields, softer breath sounds
are heard. Which of the following best describes these findings?
A. Normal
B. Asthmatic
C. Atelectasis
D. Foreign body
Explanations
(c) A. Bronchial breath sounds are normally heard near the sternum and vesicular breath sounds are heard over
the periphery of the lungs in a healthy, normal patient.
(u) B. Breath sounds in an asthmatic patient are usually obscured by wheezing.
(u) C. Breath sounds are usually absent over an area of atelectasis.
(u) D. Foreign body aspiration can present with stridor, wheezing or decreased breath sounds depending on where
it has lodged.
Ref: (1)

77. Diagnosis/Obstetrics/Gynecology
A 26 year-old gravida 0 sexually active female presents to the emergency room complaining of colicky pain in her
lower abdomen for the past 12 hours. She passed out earlier in the day while trying to have a bowel movement. Her
last menstrual period was 6 weeks ago. She has noted vaginal spotting over the last 24 hours. Vital signs show Temp
37 degrees C, BP 96/60mmHg, P 110, R 16, Oxygen Sat. 98%. Abdominal exam is positive for distension and
tenderness. Bowel sounds are decreased. Pelvic exam shows cervical motion and adnexal tenderness. Which of the
following is the most likely diagnosis?
A. Ectopic pregnancy
B. Appendicitis
C. Crohn's disease
D. Pelvic inflammatory disease
Explanations
(c) A. High suspicion for ectopic pregnancy should be maintained when any possible pregnant woman presents
with vaginal bleeding or abdominal pain.

33
(u) B. Appendicitis presents with nausea, vomiting and periumbilical pain that moves to the right lower quadrant of
the abdomen.
(u) C. Crohn's disease is more common in women and may present with an acute abdomen. However, pelvic
examination would be normal.
(u) D. In pelvic inflammatory disease the temperature is usually above 38 degrees C and pelvic pain usually
follows onset of cessation of menses.
Ref: (8)

78. Clinical Therapeutics/Cardiology


When utilizing medical treatment for hypertension, which of the following classes of medications should be used with
caution in those with elevated potassium levels or intrinsic renal disease?
A. Beta blockers
B. Calcium channel blockers
C. Central alpha agonists
D. ACE inhibitors
Explanations
(u) A. Beta blockers primarily work by decreasing cardiac contraction and slowing the heart rate. There is no
association with hyperkalemia.
(u) B. Calcium channel blockers (especially the dihydropyridines) act as vasodilators with some effect at lessening
cardiac contraction. Their use is not associated with hyperkalemia.
(u) C. Central alpha agonists stimulate the alpha receptors in the brain resulting in decreased vessel wall pressures.
This process is responsible for causing a lowering of the blood pressure. There is little effect on the kidney and
these agents do not cause hyperkalemia.
(c) D. ACE inhibitors are recognized as valuable agents in the management of hypertension but they may result in
hyperkalemia in patients with intrinsic renal disease. They should be used with caution in patients who are
taking potassium-sparing diuretics. These agents reduce glomerular filtration pressure resulting in retention of
potassium since less filtration occurs in the kidney.
Ref: (28)

79. Scientific Concepts/Gastrointestinal/Nutritional


A 40 year-old male presents with several months of abdominal pain and nausea. Endoscopy reveals an irregular 1 x
2 cm area of loss of rugal folds near the antrum. Biopsies show extensive mucosal and submucosal infiltration by B-
lymphocytes. The patient is given appropriate treatment and the lesion resolves. Which infectious agent is most likely
to be associated with these findings?
A. Aspergillus fumigatus
B. Helicobacter pylori
C. Human papilloma virus
D. Salmonella typhi
Explanations
(u) A. A. fumigatus would be associated with pulmonary, not gastrointestinal disease.
(c) B. H. Pylori causes gastric mucosal inflammation with PMN's and lymphocytes. Infection causes nausea and
abdominal pain. Inflammation may be confined to the superficial gastric epithelium or may extend deeper
resulting in varying degrees of gland atrophy. Eradication of H. Pylori with appropriate therapy leads to
resolution of the chronic gastritis.
(u) C. Human papilloma virus is primarily a urogenital disorder.
(u) D. Salmonella typhi infection would present with fevers, malaise, vomiting, and other symptoms. This diagnosis is
best made with blood cultures.
Ref: (28)

34
80. Clinical Intervention/Infectious Diseases
A 3 year-old presents with profuse watery diarrhea for the past three days. The child vomited twice yesterday, but not
today. On examination, the child is febrile, with pulse of 142, respiratory rate of 18, and blood pressure of 60/40
mmHg. The child is alert and responsive, with no focal findings. Which of the following is the most appropriate
intervention?
A. Antibiotic therapy
B. Loperamide (Imodium)
C. 3% normal saline IV infusion
D. Oral rehydration
Explanations
(u) A. In the US, infectious gastroenteritis is most frequently due to a virus. Antibiotic therapy may be second-line in
cases where the causative organism is bacterial, is identified, and symptoms continue.
(u) B. Loperamide may lead to toxic megacolon.
(h) C. 3% normal saline infusion may cause hypernatremia and central pontine myelinosis.
(c) D. The goal of therapy for a child with severe gastroenteritis and dehydration is to restore fluid loss. Oral
rehydration with an appropriate electrolyte solution is the best option if the child is not actively vomiting and is
alert enough to take oral fluids.
Ref: (13)

81. Health Maintenance/ENT/Ophthalmology


Which of the following is considered a risk factor for retinopathy of prematurity?
A. Maternal rubella infection
B. Maternal alcohol abuse
C. Low birth weight
D. Family history of retinal detachment
Explanations
(u) A. While maternal rubella infection is a risk factor for ocular disease in the newborn, it is not a specific risk factor
for retinopathy of prematurity.
(u) B. Maternal alcohol use is associated with the development of fetal alcohol syndrome which includes craniofacial
abnormalities, but does not include increased risk of retinopathy of prematurity.
(c) C. Risk factors for retinopathy of prematurity include low birth weight, perinatal oxygen therapy, prematurity and
sepsis.
(u) D. If retinopathy of prematurity is not treated, retinal detachment causing blindness may result, but a family history
of retinal detachment is not considered a risk factor for the development of retinopathy of prematurity.
Ref: (13)

82. Clinical Therapeutics/Orthopedics/Rheumatology


Which of the following medications inhibits prostaglandin synthesis in a patient with rheumatoid arthritis?
A. Methotrexate
B. Infliximab (Remicade)
C. Probenecid (Benemid)
D. Aspirin
Explanations
(u) A. Methotrexate inhibits the enzyme dihydrofolate reductase.
(u) B. Infliximab (Remicade) neutralizes cytokine tumor necrosing factor.
(u) C. Probenecid blocks the tubular reabsorption of filtered urate and is used to reduce serum uric acid.
(c) D. Salicylates inhibit the enzymatic production of prostaglandins by inhibiting cyclooxygenase.
Ref: (26)

35
83. Diagnosis/Psychiatry/Behavioral Medicine
A 32 year-old female presents to the office with the complaint of worry which she can not control for the last six
months. She tells you that she has symptoms at least four times per week consisting of sleep disturbances, difficulty
concentrating and irritability. What is the most likely diagnosis?
A. Panic disorder
B. Generalized anxiety disorder
C. Posttraumatic stress disorder
D. Obsessive-compulsive disorder
Explanations
(u) A. Panic disorder typically involves certain situations or phobias rather than generalized symptoms.
(c) B. A patient needs to have symptoms for more days than not for six months or more, need 3 of 6 symptoms to
diagnose generalized anxiety disorder.
(u) C. Posttraumatic stress disorder is characterized by development of symptoms after exposure to traumatic
events.
(u) D. In obsessive-compulsive disorder patients have symptoms of intrusive thoughts, rituals, preoccupations, and
compulsions.
Ref: (14)

84. Diagnostic Studies/Pulmonology


A 26 year-old man presents to the emergency room complaining of shortness of breath, palpitations, and tingling of
the lips and fingers. He appears anxious and describes a sensation of impending doom. His ECG and plain chest
radiograph are normal. Which of the following arterial blood gas findings would you expect in this patient?
A. pH 7.32
pCO2 49 mm Hg
bicarbonate 24 mEq/L
B. pH 7.40
pCO2 40 mm Hg
bicarbonate 25 mEq/L
C. pH 7.50
pCO2 23 mm Hg
bicarbonate 21 mEq/L
D. pH 7.52
pCO2 40 mm Hg
bicarbonate 44 mEq/L
Explanations
(u) A. These values are consistent with a respiratory acidosis.
(u) B. These values fall within the normal range.
(c) C. This patient is suffering an acute anxiety attack and resultant hyperventilation and respiratory alkalosis. Labs
show alkaline pH, depressed pCO2 and low to normal bicarbonate.
(u) D. These values suggest metabolic alkalosis.
Ref: (29)

85. Clinical Intervention/Cardiology


A 60 year-old male has unstable angina, but is otherwise healthy. A 90% lesion is found in the left main coronary
artery. Which of the following interventions is most appropriate?
A. Thrombolysis with t-PA
B. Medical management with nitrates
C. Coronary artery bypass graft (CABG)
D. Percutaneous transluminal coronary angioplasty
Explanations
(u) A. Thrombolysis is recommended in acute embolic occlusion, not chronic.
(u) B. Medical management is appropriate only for patients who are not surgical candidates.
(c) C. CABG is indicated in patients with stenosis of the left main coronary artery and those with three-vessel

36
coronary artery disease.
(u) D. Percutaneous transluminal coronary angioplasty is not the management of choice in left mainstem artery
disease because of increased potential complications and mortality.
Ref: (30)

86. History & Physical/Endocrinology


Which of the following descriptions is most typical of a patient with acromegaly?
A. Mask-like face with decreased blinking and oily skin
B. Non-pitting facial edema with dry course hair and dry skin
C. Round face and red cheeks with hirsutism
D. Coarsened facial features with prognathism and prominent brow
Explanations
(u) A. Mask-like face with decreased blinking and oily skin is usually seen in a Parkinson's disease.
(u) B. Non-pitting facial edema with dry course hair and dry skin is commonly seen with myxedema.
(u) C. Round face and red cheeks with hirsutism is commonly seen in Cushing's syndrome
(c) D. Coarsened facial features with prognathism and prominent brow are typical findings for acromegaly.
Ref: (3)

87. Health Maintenance/Gastrointestinal/Nutritional


The birth weight of an infant has usually tripled by
A. the second and third month.
B. the fourth and fifth month.
C. the seventh and eighth month.
D. one year.
Explanations
(u) A. See D for explanation.
(u) B. See D for explanation.
(u) C. See D for explanation.
(c) D. A child triples his/her birthweight by one year.
Ref: (13)

88. Diagnosis/ENT/Ophthalmology
A patient presents with complaint of sudden onset of recurrent episodic vertigo for one week that happens when
rolling onto the left side. The patient states that this sensation lasts approximately 30 seconds and then goes away.
The patient admits to associated nausea. The patient denies associated hearing difficulties or tinnitus. Which of the
following is the most likely diagnosis?
A. Benign positional vertigo
B. Mnire's disease
C. Acoustic neuroma
D. Vestibular neuronitis
Explanations
(c) A. Benign positional vertigo is characterized by the sudden onset of vertigo when rolling onto the affected side
or tilting the head up. The typical duration is less than a minute. There can be associated nausea and
vomiting. There is no impact on hearing and no associated tinnitus.
(u) B. Mnire's disease is characterized by a sudden onset of vertigo that lasts several hours to more than a day.
Patients typically have sensorineural hearing loss and tinnitus.
(u) C. Acoustic neuroma is characterized by an insidious onset of vertigo with impaired unilateral hearing and the
presence of tinnitus.
(u) D. Vestibular neuronitis (acute labyrinthitis) has a sudden onset of vertigo lasting hours to two weeks. There is
no hearing impairment or tinnitus.
Ref: (3)

37
89. Diagnostic Studies/Neurology
A 21 year-old male college student is admitted to the hospital with suspected meningitis. A lumbar puncture is
performed. The results of the cerebrospinal fluid (CSF) analysis reveals an elevated white blood cell count of
5,000/mcL with over 90% neutrophils, a decreased glucose level of 35 mg/dL, and elevated protein level of 150
mg/dL. What is the most likely diagnosis based on these results?
A. Bacterial meningitis
B. Viral meningitis
C. Fungal meningitis
D. Tuberculous meningitis
Explanations
(c) A. CSF results with bacterial meningitis reveal an elevated white count with predominance of neutrophils, a low
glucose, and an elevated protein level.
(u) B. CSF values in patients with viral meningitis are lymphocytic pleocytosis with normal glucose and normal or
slightly elevated protein.
(u) C. CSF findings in fungal meningitis include lymphocytic pleocytosis, elevated protein, and decreased glucose.
(u) D. CSF findings with TB meningitis reveals elevated pressure, lymphocytic pleocytosis, elevated protein, and
decreased glucose.
Ref: (1)

90. Scientific Concepts/Cardiology


Which of the following pathogens has been linked with the development of acute myocarditis?
A. Human papilloma virus
B. Rotavirus
C. Human Herpes Virus 6
D. Coxsackie B virus
Explanations
(u) A. Human papilloma virus most commonly is associated with venereal warts and not myocarditis.
(u) B. Rotavirus is primarily responsible for acute diarrhea.
(u) C. Human Herpes Virus 6 is the causative organism for Roseola.
(c) D. Although associated with a number of infectious and systemic diseases, myocarditis is most frequently the
result of a viral infection, with Coxsackie B virus and echovirus being the most frequently implicated in the
infection.
Ref: (1)

91. Clinical Therapeutics/Urology/Renal


Which of the following medications is most likely to cause acute tubular necrosis?
A. Trimethoprim-sulfamethoxazole (Bactrim)
B. Acetaminophen
C. Cephalothin (Kefzol)
D. Gentamicin
Explanations
(u) A. See D for explanation.
(u) B. Acute renal failure can develop as a result of acetaminophen overdose however this is rare.
(u) C. Some first generation cephalosporins may cause renal insufficiency this too is rare. Aminoglycosides are still
more nephrotoxic.
(c) D. In hospitalized patients up to 25% of patients receiving aminoglycosides sustain some degree of acute tubular
necrosis. Gentamicin is one of the most toxic aminoglycosides, streptomycin is the least nephrotoxic of the
aminoglycosides.

Ref: (28)

38
92. Diagnosis/Pulmonology
A 32 week preterm infant has an APGAR score of 9 at 5 minutes. Thirty minutes after delivery, tachypnea,
retractions, and expiratory grunting are noted. Cyanosis and dyspnea appear with little response to oxygen. Physical
examination reveals poor air movement bilaterally. A chest x-ray reveals air bronchograms and a fine reticular
granular pattern. Which of the following conditions should be suspected?
A. Atelectasis
B. Diaphragmatic hernia
C. Respiratory distress syndrome
D. Pneumothorax
Explanations
(u) A. Small areas of atelectasis usually are asymptomatic. While larger areas may present with similar clinical
findings, the chest x-ray findings are not consistent with atelectasis.
(u) B. Chest x-ray in a patient with a diaphragmatic hernia would not show a fine reticular granular pattern.
(c) C. Clinical findings of increasing cyanosis unresponsive to oxygen therapy and the characteristic x-ray findings
are most consistent with respiratory distress syndrome.
(u) D. Chest x-ray in a patient with a pneumothorax would not show a fine reticular granular pattern.
Ref: (5)

93. History & Physical/Orthopedics/Rheumatology


Which of the following mechanisms of action is most commonly associated with meniscal tears?
A. Hyperextension
B. Axial loading and rotation
C. Hyperflexion
D. Valgus force to the lateral knee
Explanations
(u) A. Hyperextension injuries usually result in ACL and PCL injuries.
(c) B. Axial loading and rotation most likely result in meniscal injuries.
(u) C. Hyperflexion injuries result in PCL injuries.
(u) D. Valgus force to the lateral knee more than likely results in medial collateral ligament injuries.
Ref: (26)

94. Clinical Intervention/Obstetrics/Gynecology


A 46 year-old G4P4 African American female presents to the clinic complaining of heavy and prolonged menstrual
flow over the past 6 months. Gynecological history includes menarche age 12 and LMP 3 weeks prior. Pelvic exam
reveals a 14-week size, irregular uterus. Pelvic ultrasound shows the presence of a large intramural fibroid with
normal endometrial lining. Which of the following is the most appropriate management for this patient?
A. Oral contraceptive pill
B. Levonorgestrel-releasing IUD
C. Hysterectomy
D. Myomectomy
Explanations
(u) A. Oral contraceptive pills do not treat the leiomyomas..
(u) B. A levonorgestrel-releasing IUD works for treatment of menorrhagia related to multiple smaller leiomyomas
(c) C. Large leiomyomas are the most common indication for hysterectomy in this age group.
(u) D. Myomectomy is not an option for this large of an intramural fibroid.
Ref: (8)

39
95. Diagnostic Studies/Gastrointestinal/Nutritional
A 66 year-old female presents to your office complaining of progressive difficulty swallowing over the last 6 months.
Initially she had difficulty only with meats, but now she has dysphagia with other foods as well. Which of the following
is the most appropriate initial diagnostic study?
A. Barium swallow
B. Endoscopy
C. CT scan
D. Urea breath test
Explanations
(u) A. Barium swallow may be used in the evaluation of dysphagia, but does not allow for biopsy to be performed.
(c) B. Endoscopy is the diagnostic study of choice in a patient with progressive dysphagia because of its ability to
obtain tissue for diagnosis.
(u) C. See B for explanation.
(u) D. See B for explanation.
Ref: (28)

96. Diagnosis/Cardiology
A patient is having a routine physical examination. Funduscopic examination reveals AV narrowing and venous
nicking. The nasal border of the optic disc appears blurred. Which of the following is the most likely underlying
cause?
A. Hypertension
B. Macular degeneration
C. Retinal detachment
D. Diabetes mellitus
Explanations
(c) A. Hypertensive retinopathy may cause AV narrowing and venous nicking due to these blood vessels having
increased pressures.
(u) B. Macular degeneration is associated with the formation of Drusen and neoproliferation.
(u) C. Retinal detachment is observed funduscopically by the retina being displaced from its attachment. Patients will
present with complaints of floaters in the eye field or abrupt loss of vision if the detachment is complete.
(u) D. Diabetes mellitus is most closely associated with neovascularization and microaneurysms as its primary
manifestations.
Ref: (3)

97. Clinical Therapeutics/ENT/Ophthalmology


A 32 year-old female presents complaining of spiking fevers. She was seen four weeks ago with a complaint of left
ear pain and was treated for otitis media. She continues to have symptoms, but now has pain behind the ear. On
examination you note left post auricular tenderness and erythema. Which of the following is the treatment of choice in
this patient?
A. IV antibiotics
B. Mastoidectomy
C. IM steroids
D. Ventilating tube placement
Explanations
(c) A. IV antibiotics are the treatment of choice in a patient with mastoiditis.
(u) B. Mastoidectomy is reserved for patients with mastoiditis who fail medical therapy.
(u) C. IM steroids are not indicated in the treatment of mastoiditis.
(u) D. Ventilating tube placement is indicated in patients with auditory tube dysfunction and chronic serous otitis
media.
Ref: (28)

40
98. Health Maintenance/Dermatology
Zostavax (varicella-zoster vaccine) is contraindicated in which of the following groups of patients?
A. Chemotherapy patients
B. Adults over 60 years of age
C. Patients allergic to eggs
D. Patients who have recovered from shingles
Explanations
(c) A. Zostavax is a live attenuated vaccine and is contraindicated in patients with immunodeficiency states,
malignancy affecting the bone marrow, pregnant women, and patients taking immunosuppressive medications.
(u) B. A single dose of zoster vaccine is recommended for patients over the age of sixty without other
contraindications.
(u) C. Patients with an anaphylactic reaction to gelatin or neomycin should not receive the Zostavax vaccine.
However, egg allergy is not a contraindication to the Zostavax vaccine.
(u) D. Patients who have recovered from shingles may still receive the vaccine. This may decrease recurrent varicella
zoster and postherpetic neuralgia in these patients.
Ref: (28)

99. History & Physical/Neurology


A 75 year-old male presents to the ER with the following stroke findings: right-sided hemiparesis (face and hand more
affected than leg), homonymous hemianopsia of the right half of both visual fields, and aphasia. Where is the location
of his stroke?
A. Anterior cerebral artery
B. Middle cerebral artery
C. Posterior cerebral artery
D. Internal carotid artery
Explanations
(u) A. Patients with anterior cerebral artery stroke will have findings greater in the legs than hands.
(c) B. This case is a description of a middle cerebral artery stroke.
(u) C. Posterior cerebral artery stroke patients will have midbrain and thalamic or sensory findings.
(u) D. Internal carotid artery stroke patients will have amaurosis fugax, visual disturbances and crossed
symptoms.
Ref: (1)

100. Scientific Concepts/Pulmonology


Patients with long-term exposure to silica, coal dust, and asbestos may develop which of the following as
complications?
A. Airway hyperreactivity
B. Epithelial hyperplasia
C. Pulmonary fibrosis
D. Upper airway obstruction
Explanations
(u) A. Airway hyperreactivity is classically seen with asthma and while acute exposure to occupational mineral dusts
may cause airway hyperreactivity, this does not persist in long-term exposures.
(u) B. Epithelial hyperplasia is one of the mechanisms involved in the pathogenesis of chronic bronchitis and is not
involved in the pneumoconioses.
(c) C. The principal processes in the pathogenesis of this set of diseases is inflammation and subsequent fibrosis.
(u) D. The upper airway is not involved in this disease process.
Ref: (7)

41
101. Diagnosis/Orthopedics/Rheumatology
A 14 year-old patient, who fell on his outstretched hand, complains of pain along his entire arm. There is point
tenderness and swelling over the midshaft of the radius. There is significant pain with limited flexion of the elbow joint.
An x-ray will most likely show which of the following fractures?
A. Galeazzi's
B. Scaphoid
C. Colles'
D. Smith's
Explanations
(c) A. Galeazzi's fracture/dislocation involves a fracture of the mid or distal radial shaft with distal radioulnar joint
dislocation.
(u) B. A scaphoid fracture is a fracture of the scaphoid bone and would not cause pain in the elbow joint.
(u) C. A Colles' fracture of the distal radius has a characteristic "silver fork" deformity, but does not involve the
elbow joint.
(u) D. A Smith's fracture is the reverse of a Colles' fracture, with volar angulation of the distal radius, but does not
involve the elbow joint.
Ref: (29)

102. Diagnostic Studies/Pulmonology


You are evaluating a patient whom you suspect has asthma. You perform spirometry before and after administration
of an inhaled short-acting bronchodilator. After administration of the bronchodilator, which of the following spirometry
results would suggest reversibility?
A. Decrease in FEV1
B. Increase in FEV1
C. Decrease in FVC
D. Increase in FVC
Explanations
(u) A. See B for explanation.
(c) B. In asthma, the airway obstruction should be at least partially relieved be a short-acting bronchodilator. This
would be reflected in an increased forced expiratory volume in 1 second (FEV1).
(u) C. The forced vital capacity (FVC) is not a function of obstruction and is generally normal in early mild asthma or
lower than expected in severe or long-standing asthma. Either way, it is not expected to change with
administration of a short-acting bronchodilator.
(u) D. See C for explanation.
Ref: (7 )

103. Clinical Therapeutics/Gastrointestinal/Nutritional


Which of the following is considered to be the treatment of choice for the pruritus that occurs with primary biliary
cirrhosis?
A. Colchicine (Colzalide)
B. Atorvastatin (Lipitor)
C. Cholestyramine (Questran)
D. Enalapril (Vasotec)
Explanations
(u) A. Colchicine is used to improve the biochemical abnormalities which may slow the progression of the disease.
(u) B. Statins have a role in the management of hyperlipidemia but are not effective in this disease.
(c) C. Cholestyramine, a bile salt sequestrant, is able to decrease the pruritus that occurs from the bile stasis and
granulomas.
(u) D. ACE inhibitors are effective as antihypertensives and in preserving renal function in those with proteinuria, but
they have no role in the management of primary biliary cirrhosis.
Ref: (7)

42
104. Clinical Intervention/Psychiatry/Behavioral Medicine
A 35 year-old male patient comes back to the office for a follow-up visit. He remarks that after 5 weeks on fluoxetine
(Prozac) 20 mg per day, he still feels depressed but he denies suicidal ideations. What should you do to help this
patient?
A. Switch to tricyclic antidepressant
B. Increase the dose of fluoxetine
C. Switch the patient to divalproex (Depakote)
D. Admit the patient to the behavioral health unit
Explanations
(u) A. Tricyclics are second line in the treatment of depression.
(c) B. An antidepressant should be raised to the recommended level and maintained at that level for 4-5 weeks,
this patient was on too low of a dose.
(u) C. Divalproex is a drug that is used to treat seizures and bipolar disorder.
(u) D. This patient denies suicidal ideations, and is not in need of an admission to the behavioral health unit.
Ref: (14)

105. Scientific Concepts/Endocrinology


Which of the following results from hypersecretion of growth hormone in a 27 year-old patient?
A. Addison's disease
B. Myxedema
C. Acromegaly
D. Cushing's syndrome
Explanations
(u) A. This is a disease of low cortisol production by the adrenal cortices.
(u) B. This occurs secondary to severe deficiency of T3 and T4.
(c) C. This disease results from excessive growth hormone production generated by the anterior pituitary or an
ectopic source.
(u) D. Excessive cortisol production is the hallmark of this process.
Ref: (7)

106. History & Physical/Cardiology


A 33 year-old female presents to the office with a complaint of palpitations. There is no history of any significant heart
disease in the past and her symptoms begin and end abruptly on their own. Which of the following is the most likely
explanation for these symptoms?
A. Atrial fibrillation
B. Atrial flutter
C. Benign supraventricular tachycardia
D. Sinus tachycardia
Explanations
(u) A. Atrial fibrillation, the most common cause of sustained irregular heart rates, tends to present in an older
population or in patients that have underlying heart disease such as mitral stenosis.
(u) B. Atrial flutter is not commonly seen in young patients without underlying heart disease although it does tend to
be intermittent in nature when it does occur.
(c) C. Benign supraventricular tachycardia tends to occur in a young patient without pre-existing heart disease.
Symptoms begin and end abruptly without therapy and occurrences happen only intermittently. It is the most
likely diagnosis in this setting.
(u) D. Sinus tachycardia may occur in a young patient without underlying heart disease but the palpitations and
tachycardia tends to start and stop gradually rather than abruptly.
Ref: (7)

43
107. Diagnosis/ENT/Ophthalmology
A 66 year-old male presents complaining of 6 month history of progressive blurred vision without associated pain. On
examination there is no erythema or injection of the sclera. On funduscopic examination there is an absent red reflex
and a cloudy lens. Which of the following is the most likely diagnosis?
A. Retinal detachment
B. Chronic glaucoma
C. Age-related macular degeneration
D. Cataract
Explanations
(u) A. In retinal detachment the retina is seen hanging in the vitreous like a gray cloud.
(u) B. In chronic glaucoma there will be slight cupping of the optic disc observed.
(u) C. Findings in age-related macular degeneration include drusen, degenerative changes in retinal pigmentation,
and subretinal neovascular membrane changes.
(c) D. Cataracts present with blurred vision that progress over months to years. On examination the red reflex
becomes increasingly difficult to visualize until it is finally absent and the pupil is white.
Ref: (28)

108. Clinical Intervention/Pulmonology


A 32 year-old male with a history of Tetralogy of Fallot with poor right ventricular function presents for evaluation of
sleep apnea. There is no evidence of deviated septum. Polysomnography reveals apneic episodes of 60 seconds in
duration. Oxygen saturation falls to low levels. Which of the following is the first-line treatment in the management of
this patient?
A. Uvulopalatopharyngoplasty
B. Nasal septoplasty
C. Continuous positive airway pressure
D. Antidepressants and oxygen
Explanations
(u) A. Uvulopalatopharyngoplasty (UVVP) is a surgical procedure with resection of pharyngeal soft tissue and partial
amputation of the soft palate and uvula. It is a procedure of last resort in most cases of sleep apnea.
(u) B. Nasal septoplasty is performed if gross anatomic nasal septal deformity is present.
(c) C. Continuous positive airway pressure prevents hypoxemia and maintains patency of the airway.
(u) D. Antidepressants have no role in the treatment of sleep apnea.
Ref: (28)

109. Diagnostic Studies/Obstetrics/Gynecology


A 21 year-old obese woman complains of menstrual irregularity since menarche at age 17. She is 5'5" and weighs
180 pounds. Exam of her face reveals excessive hair growth as well as acne. Her abdomen shows midline hair
growth and truncal obesity. A previous pelvic ultrasound shows many small fluid filled ovarian cysts bilaterally. Which
of the following is the most appropriate diagnostic study to make the initial diagnosis in this patient?
A. Prolactin level
B. Endometrial biopsy
C. Free testosterone
D. Thyroid stimulating hormone
Explanations
(u) A. Prolactin level will not be elevated in a patient with polycystic ovarian syndrome.
(u) B. Endometrial hyperplasia occurs secondary to anovulation, endometrial biopsy is mandatory for follow-up
management but is not indicated at diagnosis.
(c) C. Hyperandrogenism, as evidenced by elevated free testosterone, supports the diagnosis of polycystic
ovarian syndrome.
(u) D. An increase of TSH is suggestive of hypothyroidism.
Ref: (8)

44
110. Clinical Therapeutics/Orthopedics/Rheumatology
Which of the following medications used to treat rheumatoid arthritis is contraindicated in patients with chronic
hepatitis?
A. Sulfasalazine
B. Methotrexate
C. Minocycline
D. Infliximab
Explanations
(u) A. Sulfasalazine is a second line medication that can cause neutropenia and thrombocytopenia.
(c) B. Methotrexate is contraindicated in patients with chronic hepatitis.
(u) C. Minocycline is used for early rheumatoid arthritis with minimal adverse effects.
(u) D. Infliximab is a tumor necrosing factor inhibitor and should be used cautiously in patients with heart failure.
Ref: (28)

111. History & Physical/Urology/Renal


A 22 year-old male presents to the clinic complaining of scrotal pain that radiates into the groin. Patient admits to
being a weightlifter and was lifting 24 hours prior to this pain developing into the scrotum. The patient admits to being
sexually active with only his male partner. Examination reveals a reddened scrotum and it is difficult to distinguish the
epididymis from the testes on the right side. Elevation of the right testicle brings relief of the pain. This is known as a
positive
A. Prehn's sign.
B. Cullen's sign.
C. Rovsing's sign.
D. Murphy's sign.
Explanations
(c) A. Prehn's sign is seen in epididymitis when elevation of the scrotum with the affected epididymis to the level of
the symphysis pubis brings relief from the pain.
(u) B. Cullen's sign is a bluish discoloration of the umbilicus resulting from hematoperitoneum.
(u) C. Rovsing's sign is seen in appendicitis when pressure on the left quadrant produces pain in the right lower
quadrant.
(u) D. Murphy's sign is seen in acute cholecystitis with a sharp increase in tenderness when the gall bladder
touches the examining hand causing a stop of the inspiratory effort.

Ref: (28)

112. Diagnosis/Cardiology
A patient's EKG reveals widened P waves in lead II and large negative deflection of the P wave in lead V1. Which of
the following is the most likely underlying cause for this?
A. Right atrial enlargement
B. Left atrial enlargement
C. Right ventricular hypertrophy
D. Left ventricular hypertrophy
Explanations
(u) A. Right atrial enlargement is found on EKG with peaked P waves in lead II and a large positive deflection on
the initial P wave in lead V1.
(c) B. Wide P waves in lead II and a deep negative deflection in lead V1 is due to P-mitrale which is caused by left
atrial enlargement.
(u) C. Right ventricular hypertrophy is noted on the EKG by having a large R wave in lead AvR along with a deep S
wave in leads V5 or V6.
(u) D. On an EKG, left ventricular hypertrophy is associated with tall R waves in leads V5 and V6, deep S waves in
AvR and V1, and tall R waves in AvL and AvF.
Ref: (7)

45
113. Scientific Concepts/Gastrointestinal/Nutritional
A patient diagnosed with Barrett's esophagus is at an increased risk for the development of what type of cancer?
A. Squamous cell
B. Transitional cell
C. Adenocarcinoma
D. Atypical carcinoid
Explanations
(u) A. Squamous cell is not typical for esophageal cancer.
(u) B. Transitional cell is a cancer of the bladder.
(c) C. The most serious complication of Barrett's esophagus is esophageal adenocarcinoma.
(u) D. Atypical carcinoid is more typical of lung cancer.
Ref: (7)

114. Diagnostic Studies/Endocrinology


A 60 year-old male presents with a two day history of acute, right great toe pain with no trauma or change of
activities. He states he is healthy but started a high protein diet three weeks ago to try and lose weight. Examination
reveals an erythematous, severely swollen and highly sensitive right great toe with limited range of motion at the
metatarsal phalangeal joint. No other joint abnormalities are noted. Which of the following is the most appropriate to
confirm the diagnosis?
A. Complete blood count
B. Joint fluid analysis
C. Foot radiograph
D. Rheumatoid factor
Explanations
(u) A. Normal to nonspecific findings will occur on CBC with gouty arthritis lessening this tests clinical utility.
(c) B. Joint fluid analysis is needed to confirm the diagnosis of gouty arthritis and rule out other diagnoses such as
a septic joint, psoriatic arthritis and other crystalline-associated arthritides.
(u) C. Radiographs would only show clinically obvious swelling in cases of acute gouty arthritis.
(u) D. Rheumatoid factor is not useful in diagnosing this non-autoimmune process.
Ref: (7)

115. History & Physical/Dermatology


In a patient with dyshidrotic eczema on which area of the body would associated vesicles be found?
A. Axillae
B. Intertriginous areas
C. Palms
D. Upper back
Explanations
(u) A. See C for explanation.
(u) B. See C for explanation.
(c) C. Vesicles associated with dyshidrotic eczema are most commonly found on the palms with the dorsum of the
hands spared. The soles may also be affected in a similar fashion.
(u) D. See C for explanation.
Ref: (28)

46
116. Clinical Therapeutics/Neurology
What is the recommended treatment for absence (petit mal) seizures?
A. Phenytoin (Dilantin)
B. Carbamazepine (Tegretol)
C. Ethosuximide (Zarontin)
D. Gabapentin (Neurontin)
Explanations
(u) A. Phenytoin is used to treat tonic clonic and partial seizures.
(u) B. Carbamazepine is used to treat tonic clonic and partial seizures.
(c) C. Ethosuximide, valproic acid, and clonazepam are recommended treatments for absence seizures.
(u) D. Gabapentin is used to treat partial seizures.
Ref: (28)

117. Diagnosis/Pulmonology
A 40 year-old woman presents with 3 months of dry cough and intermittent low-grade fever. She is a non-smoker and
has no significant family history or past medical history. A purified protein derivative (PPD) test was recently
performed at work and was negative. On physical examination she is afebrile with stable vital signs. Lung
auscultation reveals crackles in bilateral upper lobes. Chest x-ray shows hilar and mediastinal adenopathy, mild
interstitial disease in the upper lung zones, and several small granulomas in both lungs. What is the most likely
diagnosis?
A. Asbestosis
B. Cryptococcosis
C. Sarcoidosis
D. Tuberculosis
Explanations
(u) A. Asbestosis typically presents as interstitial disease in the lower lungs and this patient has no known exposure
to asbestos.
(u) B. Cryptococcosis typically shows pleural-based nodules on x-ray and this patient has no known risk factors
(HIV disease, COPD, chronic steroid use).
(c) C. Sarcoidosis classically presents as a vague systemic illness with radiographic evidence of any or all of the
following: granulomas, hilar and mediastinal adenopathy and interstitial infiltrate.
(u) D. The patient's recent negative PPD makes tuberculosis unlikely.
Ref: (7)

118. Clinical Intervention/ENT/Ophthalmology


A 14 year-old male presents with complaint of worsening sore throat for two weeks. He now complains of fever,
difficulty swallowing, and difficulty opening his mouth. The patient's mother states his voice seems muffled. On
examination his left tonsil is bulging and the uvula is displaced to the right. Which of the following is the most
appropriate management?
A. Needle aspiration
B. Corticosteroid administration
C. Nebulized epinephrine administration
D. Nasotracheal intubation
Explanations
(c) A. Surgical drainage by needle aspiration and antibiotic therapy is the treatment of choice for peritonsillar
abscess.
(u) B. Corticosteroids, nebulized epinephrine, and nasotracheal intubation are not indicated in the treatment of
peritonsillar abscesses.
(u) C. See B for explanation.
(u) D. See B for explanation.
Ref: (5)

47
119. Health Maintenance/Cardiology
According to the American College of Cardiology/American Heart Association classification of heart failure, which of
the following patients fits the Stage B Classification system?
A. Asymptomatic patient with no structural disease or patients who are at high risk for the development of heart
failure.
B. Asymptomatic patient with structural heart disease.
C. Symptomatic patient with structural heart disease.
D. Patients with refractory symptoms despite intervention.
Explanations
(u) A. According to the ACC/AHA 2005 guidelines, patients with risk factors for heart disease but who have yet to
develop symptoms are categorized as Stage A. These patients have hypertension and lipid disorders treated
along with lifestyle modifications.
(c) B. According to the ACC/AHA 2005 guidelines, patients with structural heart disease who have not yet
experienced symptoms are classified as Stage B. This is the initial stage in which medication therapy other
than just ACE inhibitors are recommended.
(u) C. According to the ACC/AHA 2005 guidelines, patients with symptoms and structural heart disease are
classified as Stage C.
(u) D. According to the ACC/AHA 2005 guidelines, patients with refractory symptoms are classified as Stage D.
Ref: (28)

120. Diagnostic Studies/Orthopedics/Rheumatology


Which of the following x-ray views will show the presence of a "Scotty dog" deformity seen with spondylolysis?
A. Lateral
B. Oblique
C. Anteroposterior
D. Open-mouth odontoid
Explanations
(u) A. The lateral view is the most appropriate for evaluation of the possible presence of spondylolisthesis, not
spondylolysis.
(c) B. Spondylolysis results from a defect through the pars interarticularis, which is seen as a defect in the neck of
the "Scotty dog" on the oblique view.
(u) C. Anteroposterior views will show the alignment of the spinous processes, but not a defect in the pars
interarticularis.
(u) D. The open-mouth odontoid view is used to visualize the odontoid process and the relationship between the C1
and C2 vertebrae.
Ref: (26)

121. Clinical Therapeutics/Psychiatry/Behavioral Medicine


You are treating a patient for her first episode of major depression. She has no medical problems and there is no
family history of psychiatric disorders. In addition to psychotherapy, which class of drugs should be your first choice
for the treatment of this patient?
A. Monamine oxidase inhibitors (MAOIs)
B. Tricyclic antidepressants (TCAs)
C. Selective serotonin reuptake inhibitors (SSRIs)
D. Serotonin dopamine antagonists (SDAs)
Explanations
(u) A. MAOIs should not be first choice in the treatment of this patient because of the concern for potentially lethal
hypertension and the need for a restrictive diet.
(u) B. Although, TCAs are effective in the treatment of depression, they are not considered first line and have a
significant risk of death with overdose.
(c) C. SSRIs are the first line treatment for depression because of ease of use, safety, and broad spectrum of
treatment.

48
(u) D. SDAs or atypical antidepressants are not first line treatment for depression.
Ref: (14)

122. History & Physical/Obstetrics/Gynecology


A 29 year-old female G1P1 presents to the office with a one-month history of amenorrhea and a positive home
pregnancy test. The first day of her last menstrual period (LMP) was April 4. Using Ngele's rule what is her EDC?
A. January 1
B. January 7
C. January 11
D. January 18
Explanations
(u) A. See C for explanation.
(u) B. See C for explanation.
(c) C. Ngele's rule is LMP minus 3 months plus 7 days. April 4 minus 3 months equals January 4 plus 7 days
equals January 11.
(u) D. See C for explanation.
Ref: (8)

123. Clinical Intervention/Gastrointestinal/Nutritional


An otherwise healthy 23 year-old female presents to the student health office complaining of 3 days of frequent,
watery, non-bloody stools. She denies significant abdominal pain, vomiting, fever or dark urine. Others on campus
have been seen with the same presentation this week. Initial choice of treatment includes which of the following?
A. Fluid intake and bowel rest
B. Ciprofloxacin
C. Hospital admission with IV fluids
D. Atropine
Explanations
(c) A. Most mild diarrhea will not lead to dehydration with adequate fluids and comfort with rest to the bowel.
(u) B. Empiric treatment with antibiotics is not indicated this early.
(u) C. See A for explanation.
(h) D. Atropine is contraindicated as an antidiarrheal due to the possibility of toxic megacolon.
Ref: (1)

124. Diagnosis/Urology/Renal
An elderly appearing adult male patient is transported to the emergency room with unconsciousness for an
underdetermined amount of time. There is no family and the only history is provided by the paramedics. The patient
arouses to verbal and painful stimuli. VS: T-97.0 degrees F rectally, P-52 bpm, R-10, BP-95/60 mmHg. Physical
examination is unremarkable except for ecchymosis across his extremities. A Foley catheter is inserted draining a
small amount of dark brown urine. Urine dipstick reveals 4+ positive hemoglobin and protein. Microscopic urinalysis
reveals no RBCs but many renal tubular epithelial cells and renal tubular casts. Drug screen is negative, blood
alcohol is 2.5 mg/dL, and creatinine is 4.9 mg/dL. What is the most likely diagnosis?
A. Rhabdomyolysis causing acute renal failure
B. Obstructive uropathy causing acute renal failure
C. Ethanol ingestion causing acute renal failure
D. Methanol ingestion causing acute renal failure
Explanations
(c) A. Since the patient was found unconscious for an undetermined amount of time and the blood alcohol is elevated
the patient has been in a state of prolonged immobilization resulting in muscle ischemia resulting in
myoglobinuria. This is responsible for turning the dipstick positive without the RBCs seen on the urinary

49
microscopy. The myoglobin causes an acute tubular necrosis resulting in the sloughing of the renal tubular
epithelium. Obstructive uropathy does not cause acute tubular necrosis and occurs over time. Methanol
ingestion causes visual symptoms, ethylene glycol causes renal failure.
(u) B. See A for explanation.
(u) C. See A for explanation.
(u) D. See A for explanation.
Ref: (28)

125. Clinical Therapeutics/Pulmonology


A 55 year-old man with a history of chronic bronchitis presents with two days of increased dyspnea and cough with
worsening purulent sputum production. He is currently using inhaled albuterol as needed. In addition to systemic
corticosteroids, what pharmacologic agent is warranted at this time for treatment of this patient?
A. Antibiotic
B. Inhaled corticosteroid
C. Long acting beta-agonist
D. Theophylline
Explanations
(c) A. Empiric antibiotic treatment is indicated in the treatment of acute exacerbations of COPD if there are sputum
changes suggestive of bacterial infection, such as increased quantity and purulence.
(u) B. Inhaled corticosteroids are not indicated in the management of acute exacerbations of COPD.
(u) C. Long acting beta-agonists are not indicated in the management of acute exacerbations of COPD.
(u) D. Theophylline is rarely used in the management of COPD and has no place in the management of acute
exacerbations of COPD.
Ref: (7)

126. Diagnostic Studies/Dermatology


A 26 year-old presents with two days of a generalized, non-pruritic rash with concurrent low grade fever and sore
throat. He is otherwise in good health with no history of dermatologic problems other than acne and a non-painful
ulceration at the base of his penis seven months ago that resolved spontaneously. Examination reveals a macular-
papular rash spread diffusely over the body including the palms and soles of the feet. Shallow ulcers are noted on the
buccal mucosa while the pharynx is moderately erythematous. Which of the following is the most appropriate initial
diagnostic study?
A. Tzanck smear of lesion
B. Serum FTA-Absorption test
C. Streptococcus pyogenes culture
D. Epstein-Barr IgM titer
Explanations
(u) A. A Tzanck smear is commonly used for diagnosing herpes virus infections.
(c) B. With secondary syphilis, 100% of persons test positive with serum FTA-Absorption test.
(u) C. Though some attributes of the case point to a scarlatina- like rash the prior genital lesion and palmar
involvement make this less likely.
(u) D. Epstein-Barr IgM titer is used in the diagnosis of infections such as mononucleosis which would generally not
present with this type of rash unless a penicillin had been given to the patient.
Ref: (28)

50
127. Diagnosis/Cardiology
A 15 year-old male presents acutely to the office. His legs are cool to the touch. Examination reveals that his pulses
and blood pressure are higher in the upper extremities than the lower extremities. Femoral pulses are delayed and
weakened. Which of the following is the most likely underlying diagnosis?
A. Pheochromocytoma
B. Conn's Syndrome
C. Cushing's Syndrome
D. Coarctation of the aorta
Explanations
(u) A. Pheochromocytoma is most commonly associated with palpitations and feelings of warmth along with episodic
(later sustained) hypertension.
(u) B. Conn's Syndrome, also known as primary hyperaldosteronism, is found in a patient with hypertension who has
unprovoked hypokalemia.
(u) C. Cushing's Syndrome is associated with hypertension but is associated with the typical appearance of
sustained elevated cortisol levels such as purple striae, buffalo hump, and central obesity.
(c) D. Coarctation of the aorta typically has narrowing of the aorta proximal to the left subclavian artery with resultant
high blood pressure in the upper extremities and decreased run off to the lower extremities following this
narrowed segment.
Ref: (3)

128. History & Physical/ENT/Ophthalmology


A 60 year-old patient with a history of tobacco and alcohol abuse presents for a routine physical. Which of the
following physical examination findings is suspicious for oral carcinoma?
A. Painful creamy-white patches overlying erythema
B. Red smooth surface tongue
C. Small vesicles on an erythematous base
D. White lesion that cannot be removed by rubbing
Explanations
(u) A. Oral candidiasis is characterized by painful creamy white patches overlying erythema.
(u) B. Red smooth tongue is characteristic of glossitis which is most commonly associated with nutritional
deficiencies.
(u) C. Small vesicles on erythematous bases are characteristic of herpetic stomatitis.
(c) D. Leukoplakia, premalignant lesions, are white lesions that can not be removed by rubbing.
Ref: (28)

129. Health Maintenance/Pulmonology


A 33 year-old HIV-positive woman develops an 8mm area of induration following the administration of a purified
protein derivative (PPD) test. Her chest radiograph shows no evidence of active tuberculosis (TB) infection. Which of
the following is the most appropriate clinical intervention?
A. Four-drug regimen for 4 months
B. Isoniazid with Rifampin
C. Observation only
D. Repeat PPD and chest radiograph in 3 months
Explanations
(u) A. Greater than 5 mm of induration is positive in an HIV-infected patient. A positive PPD and negative chest film
is considered latent TB infection and, while requiring treatment, does not require the full four-drug regimen.
(c) B. Isoniazid with Rifampin is recommended in HIV positive patients with a positive PPD and a negative chest x-
ray.
(u) C. Latent TB infection is associated with a risk of progression to tuberculosis and observation alone is
inadequate.
(u) D. Repeat screening is not helpful since the diagnosis of latent TB infection has already been established.
Ref: (28)

51
130. Clinical Intervention/Orthopedics/Rheumatology
An x-ray taken on a patient complaining of wrist pain after being hit by a baseball reveals a non-displaced mid-shaft
ulnar fracture. Which of the following splints is most appropriate for treatment?
A. Thumb spica
B. Sugar tong
C. Cock-up wrist
D. Short arm gutter
Explanations
(u) A. Thumb spica splints are used for scaphoid fractures.
(c) B. Sugar tong splints are best used to immobilize the elbow, wrist and forearm.
(u) C. Cock-up wrist splints may be useful in some situations unrelated to fractures, such as to immobilize the wrist
for tendinitis or to support it in the case of wrist drop due to radial nerve palsy but not to be used in wrist
fractures.
(u) D. Short arm gutter splints immobilize only the wrist and the ulnar or radial half of the hand.
Ref: (29)

131. Diagnosis/Endocrinology
A stuporous patient is brought to the emergency room with a five day history of progressive lethargy and confusion
along with polyuria and polydipsia. On examination the patient is dehydrated and is without Kussmaul respirations.
Serum glucose is 1200 mg/dL (75-110 mg/dL), serum sodium 150 mEq/L (136-146 mEq/L) serum pH is 7.5 and
serum osmolality is 320 mosm/kg (280-300 mosm/kg). Urinalysis reveals no ketones. What is the most likely
diagnosis?
A. Primary hyperaldosteronism
B. Diabetic ketoacidosis
C. Lactic acidosis
D. Hyperglycemic hyperosmolar state
Explanations
(u) A. The extreme hyperglycemia eliminates primary hyperaldosteronism as the primary diagnosis.
(u) B. The lack of acidemia, ketones and Kussmaul respiratory pattern help to eliminate this as the primary
diagnosis.
(u) C. The lack of a Kussmaul respiratory pattern greatly lessens the likelihood of lactic acidosis.
(c) D. Extreme hyperglycemia with normal pH and negative ketones are hallmark for this clinical picture.
Ref: (28)

132. History & Physical/Infectious Diseases


A two year-old male is brought into a free clinic with a fever of 102 degrees F for two days. Through an interpreter,
the mother states that they are from Romania. Examination reveals that the buccal mucosa has a grainy appearance
opposite the second molars. Which of the following is the most likely diagnosis?
A. Rheumatic fever
B. Roseola
C. Rubeola
D. Rubella
Explanations
(u) A. Rheumatic fever causes carditis, arthritis, erythema, subcutaneous nodules, and chorea.
(u) B. Roseola, caused by human herpes virus VI, causes a high fever followed by a reticular rash.
(c) C. Rubeola or measles causes Koplik spots which are described as the salt-like crystals opposite the second
molars.
(u) D. Rubella causes a macular rash starting on the face with arthralgias and low-grade fevers.
Ref: (28)

52
133. Diagnostic Studies/Cardiology
Which of the following valvular heart abnormalities will most likely be seen on echocardiography as a complication of
acute myocardial infarction?
A. Aortic stenosis
B. Aortic regurgitation
C. Mitral stenosis
D. Mitral regurgitation
Explanations
(u) A. Aortic stenosis puts additional strain on the left ventricle and contributes to a patient developing an acute
myocardial infarction and does not occur as a result of one.
(u) B. Aortic regurgitation is not a consequence of acute myocardial infarction and most commonly occurs as a result
of an incompetent valve or dilation of the proximal aorta.
(u) C. Mitral stenosis most commonly occurs as a complication of rheumatic fever and not because of an acute
myocardial infarction.
(c) D. In patients with acute myocardial infarction, echocardiogram can show the severity of mitral regurgitation and
the presence of ventricular septal defect if one is present. Acute inferior wall myocardial infarction is associated
with acute mitral regurgitation due to necrosis of the posterior papillary muscle which is supplied by the right
coronary artery.
Ref: (7)

134. Clinical Therapeutics/Obstetrics/Gynecology


A 29 year-old female has been diagnosed with infertility due to anovulation. Her provider suggests using a medication
that will block the feedback inhibition of estradiol on the hypothalamus and pituitary leading to an increase in FSH.
Which of the following medications is the most appropriate for this patient?
A. Leuprolide (Lupron)
B. Clomiphene citrate (Clomid)
C. Medroxyprogesterone acetate (Provera)
D. Metformin (Glucophage)
Explanations
(u) A. Leuprolide inhibits gonadotropin release suppressing ovarian steroidogenesis and ovulation.
(c) B. Clomiphene citrate is the agent of choice for women younger than 36 years of age who need induction of
ovulation.
(u) C. Medroxyprogesterone acetate inhibits pituitary gonadotropin release, it maintains a pregnancy; used for
secondary amenorrhea.
(u) D. Metformin decreases hepatic glucose, reduces body weight which in turn can improve ovulatory function in
women with PCOS; it is sometimes used as an adjunct with clomiphene citrate in anovulation.
Ref: (8)

135. Health Maintenance/Pulmonology


You are seeing 62 year-old African American male for health maintenance. He is a former cigarette smoker with a 40
pack-year history. He quit smoking 10 years ago. He denies cough, hemoptysis, shortness of breath, chest pain,
weight loss, or night sweats. What method of screening for lung cancer is appropriate in this patient?
A. Chest radiograph
B. Spiral CT of the chest
C. Sputum cytology
D. No screening is recommended
Explanations
(u) A. Screening chest radiographs have not been shown to improve outcomes when used to screen asymptomatic
patients.
(u) B. While CT may yet prove valuable for screening, it is cost-prohibitive and has not yet been validated as a
screening modality in asymptomatic patients.
(u) C. Sputum cytology is not an effective screening tool.

53
(c) D. No routine screening for lung cancer is recommended for asymptomatic smokers or former smokers.
Ref: (7)

136. Clinical Intervention/Urology/Renal


A 23 year-old male being treated for an acute bacterial prostatitis has been taking antibiotics for less than 24 hours.
He presents to the emergency room today with acute urinary retention for 12 hours. Which of the following is the most
appropriate next step?
A. Insert a Foley catheter.
B. Initiate diuretic therapy.
C. Schedule for cystoscopy.
D. Insert a percutaneous suprapubic tube.
Explanations
(h) A. Urethral catheterization, or any form of instrumentation is contraindicated in the presence of acute bacterial
prostatitis.
(h) B. Diuretic therapy is contraindicated in the treatment of acute urinary retention.
(u) C. See A for explanation.
(c) D. Inserting a percutaneous suprapubic tube is the treatment of choice in a patient with acute bacterial
prostatitis who develops acute urinary retention.
Ref: (33)

137. History & Physical/Gastrointestinal/Nutritional


A 72 year-old man presents with acute left lower quadrant abdominal pain. He has nausea, vomiting, and
constipation. He has a fever of 101 F and guarding and rebound tenderness in his left lower quadrant. His white
blood cell count is elevated. He has no prior history of gastrointestinal disease. Which of the following is the most
likely diagnosis?
A. Inflammatory bowel disease
B. Irritable bowel syndrome
C. Viral gastroenteritis
D. Acute diverticulitis
Explanations
(u) A. Inflammatory bowel disease typically presents in a younger population.
(u) B. Irritable bowel syndrome is not associated with nausea, vomiting and fever. It usually presents in a younger
population.
(u) C. Viral gastroenteritis typically does not localize to the left lower quadrant.
(c) D. Acute abdominal pain, fever, left lower abdominal tenderness, and leukocytosis are hallmark signs of acute
diverticulitis.
Ref: (28)

138. Diagnosis/Neurology
You are examining a patient with right-sided extremity weakness and left-sided weakness of the face. Where is the
lesion?
A. Brainstem
B. Cerebral hemisphere
C. Cerebellum
D. Basal ganglia
Explanations
(c) A. A patient with a unilateral sensory or weakness finding on one side of the body and contralateral finding of
weakness or sensory loss of the face has a brainstem lesion.
(u) B. Patients with cerebral hemisphere lesions may present with motor, sensory, visual or auditory findings
depending on the lesion.

54
(u) C. Patients with cerebellum lesions present with ataxia, intention tremor, and dysmetria.
(u) D. Patients with basal ganglia lesions present with bradykinesia, akinesia, and loss of postural reflexes.
Ref: (1)

139. Clinical Therapeutics/ENT/Ophthalmology


A 9 year-old patient presents for follow up of his allergic rhinitis symptoms. He continues to complain of nasal
congestion, sneezing, rhinorrhea, and eczema despite avoidance therapy and treatment with oral cetirizine (Zyrtec)
and nasal flunisolide (Nasarel). Examination reveals pale, boggy nasal mucosa and eczema of the face and lower
extremities. Which of the following is the most appropriate treatment at this time?
A. Ipratropium bromide
B. Montelukast
C. Immunotherapy
D. Cromolyn sodium
Explanations
(u) A. Ipratropium bromide does not alleviate the sneezing and pruritus symptoms this patient is experiencing.
(u) B. Montelukast is less effective than intranasal steroids in the management of allergic rhinitis.
(c) C. Immunotherapy is recommended in patients with severe allergic rhinitis who fail to respond to drug therapy
and allergen avoidance. This patient has failed avoidance therapy, as well as antihistamines and intranasal
corticosteroids.
(u) D. Cromolyn sodium is much less potent than intranasal steroids and will likely not improve the patient's
symptoms.
Ref: (28)

140. Diagnostic Studies/Psychiatry/Behavioral Medicine


What laboratory test should be followed routinely every six to twelve months in patients taking lithium?
A. Complete blood count
B. Calcium
C. Potassium
D. Thyroid stimulating hormone
Explanations
(u) A. See D for explanation.
(u) B. See D for explanation.
(u) C. See D for explanation.
(c) D. Lithium induces hypothyroidism because of the decrease in concentration of circulating thyroid hormones.
Ref: (14)

141. History & Physical/Cardiology


Which of the following conditions is most closely associated with an increased intensity of the P2 heart sound?
A. Atrial septal defect
B. Aortic stenosis
C. Ventricular septal defect
D. Mitral valve prolapse
Explanations
(c) A. Atrial septal defect can cause a left to right shunt with resultant increased volume in the right ventricle. The net
result of this is that the P2 heart sound will be accentuated because of the increased blood flow in the right
ventricle and increased force of contraction in the right ventricle to remove this blood. Atrial septal defect will
also cause a fixed split S2 heart sound.
(u) B. Aortic stenosis primarily affects the left ventricle and causes left ventricular hypertrophy. As the left ventricle
contracts harder against increased valvular resistance, a systolic ejection murmur occurs.

55
(u) C. Ventricular septal defect most commonly causes a holosystolic murmur as its most notable ausculatory finding.
(u) D. Mitral valve prolapse most commonly is associated with a mid-systolic click with or without a systolic heart
murmur.
Ref: (3)

142. Diagnosis/Orthopedics/Rheumatology
A 12 year-old male presents with pain in his left leg that is worse at night. Aspirin relieves the pain and the patient
denies injury. On examination, there is point tenderness over the tibia, and the patient has a slight limp that favors the
left leg. Radiographs show a 1 cm radiolucent nidus surrounded by osteosclerosis. Which of the following is the most
likely diagnosis?
A. Osteosarcoma
B. Legg-Calve-Perthes disease
C. Osgood-Schlatter disease
D. Osteoid osteoma
Explanations
(u) A. Osteosarcoma and Ewing sarcoma are malignant bone tumors that present with pain and swelling. No
improvement is noted with conservative therapy.
(u) B. Legg-Calve-Perthes disease is avascular necrosis of the hip affecting boys ages 4-10.
(u) C. Osgood-Schlatter disease is inflammation of the tibial tuberosity affecting mainly boys in the ages of 10-15.
Commonly associated bilaterally and due to jumping.
(c) D. Osteoid osteoma is a benign tumor in children age 5 to 20, presents with increasing pain, worse at night and
relieved by aspirin.
Ref: (26)

143. Clinical Therapeutics/Gastrointestinal/Nutritional


A 25 year-old man presents with odynophagia and dysphagia. On endoscopic examination, small, white, patches with
surrounding erythema of the esophagus are noted. Silver stain is positive for hyphae. The best treatment option for
this patient is
A. acyclovir (Zovirax).
B. omeprazole (Prilosec).
C. fluconazole (Diflucan).
D. penicillin G.
Explanations
(u) A. Acyclovir is an antiviral used in the treatment of herpes esophagitis.
(u) B. Omeprazole is a proton pump inhibitor used in the treatment of gastroesophageal reflux disease with
esophageal ulceration and peptic ulcer disease and is not indicated in the treatment of infectious esophagitis.
(c) C. The patient has Candida esophagitis and the treatment of choice is fluconazole.
(u) D. Penicillin G is an antibiotic and is not effective against fungal infections.
Ref: (7)

144. Clinical Intervention/Dermatology


A 28 year-old male presents with burns sustained from hot grease splashed on his left hand earlier this afternoon.
The burn extends from his palm to the volar aspect of his wrist and has an erythematous base, covered by an intact
blister. There are a few small scattered blisters over the dorsum of the left hand. Which of the following is the initial
intervention of choice?
A. Tetanus prophylaxis
B. Admission to a burn unit
C. Intravenous fluid administration
D. Debridement of blisters

56
Explanations
(c) A. Tetanus prophylaxis should be initially considered in all burn patients.
(u) B. Admission to a burn unit is not indicated for adult patients with uncomplicated partial thickness burns
covering less than 15 to 20% of total body surface area (TBSA).
(u) C. IV fluids are indicated for severe partial thickness burns covering more than 10% TBSA or in burns with
complications.
(u) D. Debridement of blisters is controversial, however blisters on the palms and soles should remain intact.
Ref: (17)

145. Clinical Therapeutics/Pulmonology


A patient is brought to the emergency room with acute onset of dyspnea and tachypnea. He has a long history of
alcoholism and was involved in a motor vehicle accident two days ago. He is hypoxic with crackles auscultated
bilaterally. Chest radiography reveals diffuse bilateral infiltrates which spare the costophrenic angle and air
bronchograms, there was no cardiomegaly or pleural effusion noted. Oxygen saturation is 70%. Which of the
following is the most important initial treatment?
A. Tracheal intubation
B. Bilateral chest tube insertion
C. Type-specific packed cells
D. Colloid solutions
Explanations
(c) A. Tracheal intubation with lowest level of PEEP is required to maintain the PaO2 above 60mmHg or SaO2
above 90% in a patient with ARDS.
(u) B. See A for explanation.
(u) C. Fluids are the preferred treatment initially for hypovolemia. Type-specific packed cells are given when the
patient's blood type is identified. Until then O negative packed cells are administered.
(u) D. Use of crystalloid solutions are preferred to avoid pulmonary edema.
Ref: (28)

146. Scientific Concepts/Neurology


You have just stuck yourself with a sharp needle. In order for you to be able to interpret this sensation, which of the
following areas must be intact?
A. Anterior spinothalamic tract, basal ganglia, and sensory cortex
B. Corticospinal tract, medulla, and basal ganglia
C. Pyramidal tract, hypothalamus, and sensory cortex
D. Lateral spinothalamic tract, thalamus, and sensory cortex
Explanations
(u) A. See D for explanation.
(u) B. See D for explanation.
(u) C. See D for explanation.
(c) D. Sensory impulses reach the sensory cortex from the spinothalamic tract or the posterior columns. Fibers
transmit this to the thalamus which sends impulses to the sensory cortex of the brain
Ref: (3)

147. Diagnosis/Obstetrics/Gynecology
A 22 year-old G1P0 female presents at 12 weeks gestation with 24 hours of vaginal bleeding. She complains of
continued cramping and bleeding requiring pad change every two hours. Vital signs are normal. Vaginal examination
shows bleeding with a dilated cervix. Which of the following is the most likely diagnosis?
A. Threatened abortion
B. Inevitable abortion
C. Incomplete abortion

57
D. Complete abortion
Explanations
(u) A. Threatened abortion (AB) implies the cervix remains closed with only slight bleeding.
(c) B. Abortion is inevitable when cervical effacement, dilatation and rupture of membranes is noted.
(u) C. Incomplete AB implies the products of conception have partially passed causing continued bleeding, more
common after 10 weeks.
(u) D. Complete AB is identified by passage of the entire conceptus.
Ref: (8)

148. Clinical Therapeutics/Cardiology


Which of the following medication classes is considered first-line therapy for patients with Raynaud's phenomenon?
A. Beta blockers
B. Calcium channel blockers
C. Central alpha agonists
D. Oral nitrates
Explanations
(u) A. Beta blockers do not cause vasodilation against this vasospasm which makes them less effective as treatment.
(c) B. Calcium channel blockers are first line therapy for patients who have uncontrolled symptoms related to
Raynaud's phenomenon. They are vasodilating agents which may play a role in preventing the vasospasm that
occurs with this disorder.
(u) C. See B for explanation.
(u) D. Second line agents for Raynaud's phenomenon may include topical but not oral nitrates.
Ref: (28)

149. Health Maintenance/Gastrointestinal/Nutritional


A 41 year-old female presents to you for medical screening advice. Her 44 year-old sister passed away recently 18
months after diagnosis of metastatic colon cancer. Which of the following is the most appropriate advice for this
patient?
A. Double contrast barium enema now and repeat every 5 years if normal
B. Rectal occult blood testing annually until age 50 then sigmoidoscopy every 3 years
C. Rectal occult blood testing annually until age 50 then screening colonoscopy every 5 years
D. Screening colonoscopy now and repeat every 3-5 years if normal

Explanations
(u) A. See D for explanation.
(u) B. See D for explanation.
(u) C. See D for explanation.
(c) D. Hereditary factors are believed to contribute to up to 30% of colorectal cancers. Relative risk is 3.8 times if the
family member's cancer was diagnosed at less than 45 years of age. Recommended screening in a single first
degree relative with colorectal cancer diagnosed before age 60 is beginning colonoscopy at age 40 or ten
years younger than age at diagnosis of youngest affected first-degree relative. Then if negative, every 5 years.
Ref: (28)

150. History & Physical/Pulmonology


A 27 year-old woman presents with one week of worsening productive cough, dyspnea, fever and malaise. Which of
the following physical examination findings would support the diagnosis of lobar pneumonia?
A. Decreased tactile fremitus
B. Egophony
C. Hyperresonance to percussion
D. Wheeze

58
Explanations
(u) A. Consolidation would increase the transmission of vocal vibrations and manifest as increased tactile fremitus.
(c) B. Transmission of vocal sounds through consolidation leads to the changes heard with egophony.
(u) C. Dullness, not hyperresonance, would be expected with consolidation.
(u) D. Wheezing is associated with narrowing of the small airways as seen in asthma.
Ref: (3)

151. Clinical Intervention/Orthopedics/Rheumatology


Which of the following interventions is initially indicated for helping to relieve the symptoms of plantar fasciitis?
A. Steroid injections
B. Short leg walking cast
C. Arch supports
D. Surgical release
Explanations
(u) A. Steroid injections can be used to relieve symptoms but may be harmful by causing rupture if given in the
plantar tendon greater than 3 or 4 times.
(u) B. Short leg walking cast may be used for severe cases refractory to initial treatment.
(c) C. Arch supports, NSAIDs and stretching exercises are the initial interventions to help relieve symptoms.
(u) D. Surgical release of the plantar tendon is usually reserved for patients who fail all therapies.
Ref: (29)

152. Diagnosis/Gastrointestinal/Nutritional
A 65 year-old homeless male with a history of pancreatitis is seen in the emergency department for vomiting, upper
abdominal pain, back pain and weakness. He is cachetic, pale and jaundiced. A 4-5 cm mass is palpable in the mid
to right hypochondrium. What is the most likely diagnosis?
A. Chronic cholecystitis
B. Carcinoma of head of pancreas
C. Fibrolipoma
D. Primary biliary cirrhosis
Explanations
(u) A. Chronic cholecystitis is not typically associated with weight loss or cachexia. There would not be a palpable
mass.
(c) B. Seventy-five percent of pancreatic cancers are in the head. Risk factors include age, tobacco use, obesity,
chronic pancreatitis, family history and previous abdominal radiation.
(u) C. Fibrolipoma may present as an abdominal mass, but would not cause weight loss and illness.
(u) D. Primary biliary cirrhosis most commonly presents with generalized urticaria and is not associated with an
abdominal mass.
Ref: (28)

153. Clinical Therapeutics/Urology/Renal


A 23 year-old male presents with symptoms of irritative urethral discharge. History reveals recent unprotected
intercourse with a new partner. A Gram stain of the urethral discharge is negative for intracellular diplococci.
Assuming no allergies, which of the following is the drug of choice?
A. Doxycycline
B. Penicillin G
C. Ceftriaxone
D. Amoxicillin

59
Explanations
(c) A. Doxycycline 100 mg orally for 7 days is indicated for the treatment of non-gonococcal urethritis.
(u) B. Penicillin G is not used in the treatment of Chlamydia urethritis.
(u) C. Ceftriaxone is the treatment of choice for uncomplicated gonorrhea.
(u) D. Amoxicillin is not used in the treatment of gonococcal or non-gonococcal urethritis.

Ref: (28)

154. Scientific Concepts/Cardiology


Elevated levels of LP(a) (Lipoprotein a) are considered to be a risk factor for coronary artery disease through which of
the following proposed mechanisms?
A. Direct inhibition of HDL
B. Increasing the formation of VLDL cholesterol
C. Competes for binding to the plasminogen receptor
D. Enhancement of naturally circulating triglycerides
Explanations
(u) A. See C for explanation.
(u) B. See C for explanation.
(c) C. Lp(a) lipoproteins are secreted by the liver, constitute 10% or less of the total plasma lipoprotein mass,
possess kringle domains homologous to plasminogen, and are associated with vascular disease risk. Having
domains homologous to plasminogen, Lp(a) will compete with actual plasminogen for its receptor sites.
Plasminogen activates plasmin, which facilitates degradation of fibrin and matrix components. The main
component of LP (a) is LDL, a known risk factor for atherosclerosis.
(u) D. See C for explanation.
Ref: (1)

155. Diagnostic Studies/Endocrinology


A 50 year-old female presents describing poor concentration, generalized fatigue, oligomenorrhea and severe acne
outbreaks over the past year. On examination she exhibits central obesity, a round face, numerous striae and
elevated blood pressure. Which of the following is the most appropriate initial diagnostic study?
A. Serum thyroid stimulating hormone
B. Dexamethasone suppression test
C. Oral glucose tolerance test
D. Antithyroperoxidase antibodies
Explanations
(u) A. Serum thyroid stimulating hormone would be more appropriate for hypo- or hyperthyroidism.
(c) B. Dexamethasone suppression test as well as midnight serum cortisol level, late night salivary cortisol level, and
urinary free cortisol used singly or in combination to diagnose Cushing's syndrome (Hypercortisolism).
(u) C. Patients with Cushing's syndrome do have issues with glucose tolerance, however oral glucose tolerance test
would not confirm the presence of a secondary cause of the intolerance.
(u) D. Antithyroperoxidase antibodies are commonly ordered when Hashimoto thyroiditis is suspected.
Ref: (28)

156. . Health Maintenance/Hematology


Adult patients with a compromised immune system should not receive which of the following immunizations?
A. Hepatitis A
B. Pneumococcal
C. Influenza
D. Varicella

60
Explanations
(u) A. See D for explanation.
(u) B. See D for explanation.
(u) C. See D for explanation.
(c) D. Varicella vaccine is a live vaccine and is contraindicated in those individuals with a compromised immune
system. Hepatitis A, pneumococcal and influenza are recommended for patients with a compromised immune
system.
Ref: (28)

157. Clinical Therapeutics/Orthopedics/Rheumatology


When injecting a corticosteroid into a joint to help relieve pain, which of the following would most commonly be mixed
in the syringe?
A. Hyaluronic acid
B. Ketorolac
C. Chondroitin sulfate
D. Lidocaine
Explanations
(u) A. Hyaluronic acid is usually injected by itself in patients with osteoarthritis.
(u) B. Ketorolac is an NSAID that is given orally and not intra-articularly.
(u) C. Chondroitin sulfate is taken orally and thought to improve symptoms in osteoarthritis.
(c) D. 1% lidocaine and corticosteroids are most commonly used together for joint injections.
Ref: (19)

158. Clinical Intervention/Pulmonology


A 4 year-old boy is sent home from day care for a severe cough following one week of cold symptoms, including
sneezing, conjunctivitis, and nocturnal cough. He presents with paroxysms of cough followed by a deep inspiration,
and occasional post-tussive emesis. During severe paroxysms, he exhibits transient cyanosis. What is the most
appropriate treatment for exposed contacts at his day care center?
A. Macrolide prophylaxis
B. Isolation
C. Observation and treatment only if symptomatic
D. Supportive care only
Explanations
(c) A. All close contacts of a patient with pertussis should be treated with macrolide prophylaxis, regardless of age,
immunization history, or symptoms.
(u) B. Isolation of contacts is impractical and unnecessary.
(u) C. Pertussis is rarely diagnosed before the paroxysmal stage, by which time exposure of contacts to the
pathogen is assured.
(u) D. While supportive care is essential in those contacts with symptoms, macrolide prophylaxis is mandatory in
all contacts to prevent further spread of the illness.
Ref: (5)

159. Clinical Therapeutics/Cardiology


Which of the following medication classes is the recommended treatment for patients who have an anterior wall
myocardial infarction with poor left ventricular function?
A. Beta blockers
B. Calcium channel blockers
C. Potassium sparing diuretics
D. ACE inhibitors

61
Explanations
(u) A. Beta blockers need to be used with caution in a patient with severe left ventricular dysfunction as they will
worsen left ventricular contractility and may make this dysfunction worse. They are used, however, in the early
stages of chronic heart failure.
(u) B. Calcium channel blockers have no proven mortality benefit in patients with myocardial infarctions and left
ventricular dysfunction.
(u) C. Although potassium sparing diuretics are part of the later stage treatment of congestive heart failure and tend
to potentiate the other therapies, they are not first-line therapy in a patient with left ventricular dysfunction.
(c) D. ACE inhibitors have been proven to be effective in the therapy of heart failure, especially in the setting of left
ventricular dysfunction. They are considered first-line therapy in patients with symptomatic left ventricular
systolic function.
Ref: (28)

160. History & Physical/Obstetrics/Gynecology


A 46 year-old woman describes her menstrual periods as regular (occurring every 30 days), prolonged, and with a
heavy flow. You document this finding as which of the following?
A. Menorrhagia
B. Oligomenorrhea
C. Metrorrhagia
D. Hypomenorrhea
Explanations
(c) A. Menorrhagia describes long and heavy flow menstrual flow.
(u) B. Oligomenorrhea describes menstrual periods that occur more than 35 days apart.
(u) C. Metrorrhagia is bleeding that occurs at any time between menstrual periods.
(u) D. Hypomenorrhea is usually light flow, sometimes only spotting.
Ref: (8)

161. Diagnosis/Dermatology
A 30 year-old patient is seen for a non-painful mass on the upper back which has grown slowly over the past year. He
denies previous trauma, drainage from the area or history of dermatologic diseases. Examination reveals a four
centimeter firm, but highly mobile subcutaneous mass with no overlying skin discoloration or punctum with drainage.
Which of the following is the most likely diagnosis?
A. Epidermoid cyst
B. Hemangioma
C. Keratoacanthoma
D. Lipoma
Explanations
(u) A. An epidermoid cyst is a lesion of the dermis, usually with periodic expression of sebaceous material.
(u) B. A hemangioma is typically a discolored elevated bed/mass of vessels penetrating to or through the
epidermis.
(u) C. Keratoacanthoma is rapid in its development, more common in the elderly and confined to the dermis.
Resembles squamous cell carcinoma.
(c) D. A lipoma is a benign fatty tumor usually developing slowly in the subcutaneous layer and generally
remaining small though can become very large.
Ref: (10)

62
162. Diagnostic Studies/Urology/Renal
A 29 year-old male is involved in a motor vehicle crash. On the secondary survey it is noted that there is blood at the
meatus and the patient is suspected of having a pelvic fracture. The patient is otherwise stable. Which of the
following tests should be done to evaluate the urinary system?
A. Voiding cystourethrogram (VCUG)
B. Intravenous pyelogram (IVP)
C. Urethrogram
D. Renal arteriography
Explanations
(u) A. A VCUG is done to evaluate urinary reflux in children.
(u) B. An IVP can be done as part of an evaluation for hematuria however it is rarely used today.
(c) C. A urethrogram is the only procedure that should be done to evaluate this type of injury as urethral integrity may
have been compromised secondary to the pelvic fracture. Blood at the meatus is the most important finding of
suspected urethral injury.
(u) D. Renal arteriography does not have any place in this scenario as there is no evidence of a renal injury.
Ref: (33)

163. Clinical Therapeutics/Orthopedics/Rheumatology


A 35 year-old female presents with a long standing complaint of dry, scratchy eyes and dry mouth. She also reports
dyspareunia. Labs demonstrate a positive anti-nuclear antibody and Anti-La antibodies. The patient has a prolonged
Schirmer test. Which of the following classes of medicines should be avoided in treating this condition?
A. Antimalarials
B. NSAIDs
C. Glucocorticoids
D. Anticholinergics
Explanations
(u) A. See D for explanation.
(u) B. See D for explanation.
(u) C. See D for explanation.
(c) D. Anticholinergics should be avoided in a patient with Sjogrens syndrome. All the other medications may be
used to treat the systemic symptoms of Sjogrens.
Ref: (1)

164. History & Physical/Neurology


A 64 year-old right-handed woman presents to the emergency room. The patient is pleasant and cooperative, but you
note that the left side of her mouth has little movement as she talks, resulting in some dysarthric speech. On physical
examination, the left side of her mouth droops, eyebrows raise symmetrically, frown is symmetric, and eyes close but
left offers little resistance to opening. You suspect which of the following?
A. Upper motor neuron damage to CN VII (central facial palsy) in left hemisphere
B. Upper motor neuron damage to CN VII (central facial palsy) in right hemisphere
C. Lower motor neuron damage to CN VII (Bell's palsy) on the left
D. Lower motor neuron damage to CN VII (Bell's palsy) on the right
Explanations
(u) A. See B for explanation.
(c) B. The upper face is controlled by pathways from both sides of the face. Upper motor neuron lesions cross,
findings on the left are due to lesions on the right. Lower motor neuron lesions of the face are ipsilateral.
(u) C. See B for explanation.
(u) D. See B for explanation.
Ref: (3)

63
165. Diagnosis/Cardiology
The 35 year-old patient presents after a syncopal episode while throwing a football with his son. Examination reveals
regular heart rate and EKG is normal. There were no symptoms prior to the episode. Right radial pulse is decreased.
Which of the following is the most likely explanation for the syncope?
A. Carotid sinus hypersensitivity
B. Vasovagal episode
C. Cardiac dysrhythmia
D. Subclavian steal syndrome
Explanations
(u) A. Carotid sinus hypersensitivity typically presents with syncope that is related to turning of the head (such as
backing a car out of a driveway) or from tight collars.
(u) B. Vasovagal episodes may result in syncope but there would not be abnormal pulse findings.
(u) C. Cardiac dysrhythmia may occur in the setting of exercise but this would not produce a decreased pulse on the
affected side.
(c) D. Subclavian steal syndrome occurs if the subclavian artery is occluded proximal to the origin of the vertebral
artery which results in reversal in the direction of blood flow in the ipsilateral vertebral artery. Exercise of the
ipsilateral arm may increase demand on the vertebral flow which produces a "subclavian steal".
Ref: (28)

166. Clinical Intervention/Gastrointestinal/Nutritional


Which of the following is the treatment of choice for patients with celiac disease?
A. Gluten-free diet
B. Small bowel resection
C. Clindamycin
D. Whipple procedure
Explanations
(c) A. Removal of all gluten from the diet is essential to therapy in celiac disease.
(u) B. Celiac disease is a dietary disorder that resolves with gluten free diet. Small bowel resection is not indicated.
(u) C. Antibiotics are not therapeutic for celiac disease.
(u) D. Whipple procedure is used in the treatment of pancreatic carcinoma and is not indicated in the management of
celiac disease.
Ref: (28)

167. Health Maintenance/Psychiatry/Behavioral Medicine


Who is the most likely adult to sexually abuse a child?
A. School teacher
B. Friend of the family
C. Scout leader
D. Family member
Explanations
(u) A. See D for explanation.
(u) B. See D for explanation.
(u) C. See D for explanation
(c) D. Adults within the immediate or extended family perpetrate the most child sexual abuse, usually this is a trusted
member of the family.
Ref: (14)

64
168. Diagnostic Studies/ENT/Ophthalmology
An 18 year-old male who was struck in the left eye with a baseball presents with ocular pain, periorbital ecchymosis,
and restricted upward gaze. Which of the following is the diagnostic study of choice in this patient?
A. Zygomatic arch x-ray
B. CT scan of the orbit
C. Ultrasonography
D. Fluorescein staining
Explanations
(u) A. Zygomatic arch x-ray is not the diagnostic study of choice for a suspected blowout fracture.
(c) B. CT scan of the orbit is the study of choice to evaluate a suspected blowout fracture.
(u) C. Ultrasound may be helpful if there is a suspected foreign body, however it is not helpful in fracture
identification.
(u) D. Fluorescein staining is indicated in the evaluation of corneal abrasion, not orbital fracture.
Ref: (29)

169. History & Physical/Orthopedics/Rheumatology


In the neonate, unequal thigh folds may indicate which of the following?
A. Coxa vara
B. Legg-Calve-Perthes disease
C. Developmental hip dysplasia
D. Slipped capital femoral epiphysis
Explanations
(u) A. Coxa vara is a hip deformity that would present with a decrease in the hip angle and a shift of the femoral shaft
medially.
(u) B. Legg-Calve-Perthes disease presents with a painless limp in children ages 4-10 due to avascular necrosis of
the femoral head.
(c) C. A dislocated hip displaces proximally in developmental hip dysplasia, causing a shortening of the leg that may
present as unequal thigh folds.
(u) D. A slipped capital femoral epiphysis is primarily an adolescent disorder with decreased range of motion in
abduction and internal rotation of the hip on physical examination.
Ref: (26)

170. Clinical Therapeutics/Pulmonology


A previously healthy 8 month-old boy is hospitalized for acute bronchiolitis. He has no known significant past medical
or family history. On admission, he exhibits nasal flaring and retractions with a respiratory rate of 68, axillary
temperature of 102.0 degrees F and O2 saturation of 86%. Which of the following medications is indicated?
A. Prednisolone
B. Oxygen
C. Ceftriaxone (Rocephin)
D. Palivizumab (Synagis)
Explanations
(u) A. Corticosteroids are not indicated for the treatment of previously healthy infants with bronchiolitis.
(c) B. Oxygen is an important supportive therapy for hypoxemic infants with bronchiolitis. Bronchodilators would
also be initiated in this patient.
(u) C. Antibiotics are not indicated in the treatment of bronchiolitis unless there is a secondary bacterial infection.
(u) D. Palivizumab is used only for prevention of RSV infection.
Ref: (5)

65
171. Scientific Concepts/Cardiology
Which of the following is the most common complication that occurs in the setting of acute pericarditis?
A. Pericardial effusion
B. Left ventricular failure
C. Superior vena cava syndrome
D. Subclavian steal syndrome
Explanations
(c) A. Accumulation of transudate, exudate or blood in the pericardial sac can occur due to pericardial inflammation.
(u) B. Patients with acute pericarditis may have problems with filling which affects the right ventricle more than the
left ventricle.
(u) C. Patients with lung malignancy may develop superior vena cava syndrome as a result of tumor invasion into the
superior vena cava.
(u) D. Patients with subclavian steal syndrome typically present with arm ischemia and syncope and is not related to
pericarditis.
Ref: (1)

172. Diagnosis/Infectious Diseases


A 19 year-old college student has a severe sore throat and a temperature of 102 F. On examination, there are
vesicles on the soft palate with some erythema. The tonsils are not enlarged. There is mild cervical lymph node
enlargement. Which of the following is the most likely diagnosis?
A. Epstein-Barr virus
B. Coxsackie virus group A
C. Streptococcus pyogenes
D. Mycoplasma pneumoniae
Explanations
(u) A. Epstein-Barr is accompanied by generalized lymphadenopathy. Exudative pharyngitis and tonsillitis may also
occur.
(c) B. Coxsackie virus group A is characterized early by petechiae or papules/vesicles on the soft palate that
become shallow ulcers in about 3 days.
(u) C. Streptococcus pyogenes produces an exudative pharyngitis.
(u) D. Mycoplasma pneumoniae is characterized by conjunctivitis, otitis media, and atypical pneumonia.
Ref: (7)

173. History & Physical/Gastrointestinal/Nutritional


A 45 year-old type 2 diabetic female with history of cholelithiasis presents to the clinic with 2-3 episodes of sudden,
severe epigastric pain that radiates to her shoulder. She has associated nausea and vomiting. Temperature is 101
degrees F and she is experiencing chills. Today her eyes appear yellow in color. Which of the following is the most
likely diagnosis for this patient
A. Postcholecystectomy syndrome
B. Cholangitis
C. Gastroesophageal reflux disease
D. Pancreatic cancer
Explanations
(u) A. The patient has no history of previous gall bladder surgery.
(c) B. Cholangitis is characterized by a history of biliary pain, fever, chills, and jaundice associated with episodes of
abdominal pain.
(u) C. Gastroesophageal reflux disease (GERD) is characterized by heartburn. Fever and jaundice are not typical
features of GERD.
(u) D. Pancreatic cancer, although a possibility, is characterized by chronic weight loss, epigastric pain radiating to
the back, and occasional jaundice. Fever and chills are not typical features.
Ref: (28)

66
174. Clinical Intervention/Endocrinology
A 32 year-old male with a history of pheochromocytoma is seen in the office. The patient is scheduled for
adrenalectomy, however has developed a throbbing headache and racing heart. Vital signs reveal pulse 126 bpm,
blood pressure 160/115 mmHg, and respiratory rate 20. The patient appears diaphoretic and anxious. Which of the
following is the most appropriate acute management in this patient?
A. Oral Phenoxybenzamine (Dibenzyline)
B. Hydrochlorothiazide (Diuril)
C. Lisinopril (Prinivil)
D. Bumetanide (Bumex)
Explanations
(c) A. Phenoxybenzamine is an alpha-blocker utilized to control hypertension in patient with a pheochromocytoma.
(u) B. Hydrochlorothiazide is not appropriate in the management of hypertension in a patient with a
pheochromocytoma.
(u) C. ACE inhibitors are not indicated in the management of hypertension in a patient with pheochromocytoma.
They may be used if blood pressure is difficult to control with alpha-blockers alone.
(u) D. Bumetanide is not appropriate in the management of hypertension in a patient with a pheochromocytoma.
Ref: (7)

175. Clinical Therapeutics/Hematology


A 21 year-old male with a diagnosis of type 1 von Willebrand disease undergoes dental extraction of his wisdom
teeth. The patient comes to the clinic with continued oozing of the dental sockets despite packing. Treatment should
begin with which of the following?
A. DDAVP
B. Factor VIII
C. vWF concentrate
D. FFP
Explanations
(c) A. DDAVP causes the release of vWF and factor VIII from storage sites significantly which is needed to complete
hemostasis. Factor VIII is indicated for patients with Hemophilia A. Fresh frozen plasma is indicated in
Coumadin overdosage and vWF concentrate is indicated in type 2 and 3 patients with von Willebrand disease.
(u) B. See A for explanation.
(u) C. See A for explanation.
(u) D. See A for explanation.
Ref: (28)

176. Diagnosis/Obstetrics/Gynecology Topic: 4d Author: Annette Larson


A 54 year-old female comes to your office with the complaint of vaginal itching. Her last menstrual period was three
years ago. On examination the patient's vulva is smooth and somewhat shiny; her vaginal mucosa is pale and thin
with a mild yellowish discharge with a pH of 6.0. The most likely cause of these symptoms is which of the following?
A. Lichen sclerosis
B. Atrophic vaginitis
C. Contact Dermatitis
D. Candidiasis
Explanations
(u) A. Lichen sclerosis is a benign, chronic inflammatory process and the most common vulvar dermatologic
disorder. During the acute phase the lesions appear red or purple and involve the non hair bearing areas of
the vulva and perianal areas. Erythema and edema of the skin occur.
(c) B. Atrophic vaginitis is typically caused by reduced estrogen levels producing intense itching and thin vaginal
mucosa with a resultant yellowish discharge that has a pH > 5.5.
(u) C. Contact dermatitis is relatively common with red, edematous skin surfaces and sometimes with vesicles and
secondary infection.
(u) D. Candidiasis produces a white curdy discharge with a pH of < 4.5.
Ref: (8)

67
177. Diagnostic Studies/Cardiology
Which diagnostic study is considered to be the strategy of choice for symptomatic patients with recurrent ischemia,
hemodynamic instability or impaired left ventricular dysfunction?
A. Stress echocardiography
B. Exercise treadmill testing
C. Coronary angiography
D. Cardiac magnetic resonance imaging
Explanations
(h) A. Stress echocardiography should not be performed in the setting of a patient who is acutely symptomatic.
(h) B. Exercise treadmill testing should not be performed in the setting of an unstable patient with ongoing cardiac
symptoms.
(c) C. Coronary or cardiac catheterization is the gold standard technique in the evaluation of patients with significant
cardiac symptoms. Anatomical information along with degree of coronary artery blockages are provided and
patients may be able to undergo coronary revascularization during or after this procedure.
(u) D. Cardiac magnetic resonance imaging has limited availability and is not part of national guidelines for
evaluation of the cardiac patient.
Ref: (28)

178. History & Physical/Psychiatry/Behavioral Medicine


On performing a mental status examination you notice that the patient tends to repeat words and phrases out of
context to your questions. This type of thought disorder is defined as which of the following?
A. Flight of ideas
B. Perseveration
C. Circumstantiality
D. Derailment
Explanations
(u) A. With flight of ideas, thoughts move from one topic to another with rapid speech.
(c) B. Definition of perseveration is the persistent repetition of words or concepts in the process of speaking. This is
often seen in cognitive disorders, schizophrenia, and other mental illness.
(u) C. Circumstantiality includes much detail, but information is not relevant.
(u) D. Derailment is speech with loose associations and sentences that do not make sense.
Ref: (14)

179. Clinical Intervention/ENT/Ophthalmology


An elderly patient with a history of hypertension presents with epistaxis. On examination you note blood from both
nares and down the posterior oropharynx. Examination of the nasal cavity with an ENT headlamp does not show an
area of bleeding. Which of the following is the treatment of choice in this patient?
A. Electrical cautery
B. Direct pressure on the nose
C. Petroleum jelly application
D. Posterior nasal packing
Explanations
(u) A. Cautery and direct pressure are effective in anterior, not posterior epistaxis.
(u) B. See A for explanation.
(u) C. Petroleum jelly application is not indicated in posterior epistaxis.
(c) D. Posterior epistaxis is more common in elderly patients, especially with hypertension. Posterior nasal
packing is the treatment of choice.
Ref: (29)

68
180. Clinical Therapeutics/Neurology
A 50 year-old female with a history of coronary artery disease presents to the office requesting medication for an
exacerbation of her migraine headaches. She complains of migraines approximately once a month. What medication
is contraindicated in this patient?
A. Sumatriptan (Imitrex)
B. Propranolol (Inderal)
C. Droperidol (Inapsine)
D. Naproxen sodium (Anaprox)
Explanations
(c) A. The triptans are contraindicated in patients with coronary artery disease or peripheral vascular disease and
should be avoided in patients with an increased risk for stroke. All other drugs listed are not contraindicated.
(u) B. See A for explanation.
(u) C. See A for explanation.
(u) D. See A for explanation.
Ref: (28)

181. Scientific Concepts/Orthopedics/Rheumatology


Which of the following is the underlying pathogen for the development of Lyme disease?
A. Borrelia burgdorferi
B. Bartonella henselae
C. Rickettsia rickettsii
D. Coxiella brunetti
Explanations
(c) A. Lyme disease is a tick-borne illness caused by the spirochete B. burgdorferi.
(u) B. Bartonella henselae is the spirochete that causes cat scratch fever.
(u) C. Rickettsia Rickettsii is the spirochete that causes Rocky Mountain Spotted Fever.
(u) D. Coxiella brunetti is the spirochete that causes Q fever.
Ref: (32)

182. History & Physical/Cardiology


Which of the following is a systemic manifestation of infective endocarditis?
A. Hemarthrosis
B. Petechiae
C. Cafe au lait spots
D. Bronzing of the skin
Explanations
(u) A. Hemarthrosis is most commonly a consequence of a clotting disorder such as hemophilia.
(c) B. Petechiae, splinter hemorrhages, Janeway lesions, and Osler's nodes are systemic manifestations of patients
who have infective endocarditis.
(u) C. Cafe au lait spots are seen in Neurofibromatosis (von Recklinghausen's syndrome).
(u) D. Bronzing of the skin is most commonly associated with hemochromatosis or Addison's disease.
Ref: (3)

69
183. Diagnostic Studies/Gastrointestinal/Nutritional
Which of the following is the most appropriate study for diagnosing Hirschsprung disease?
A. Rectal biopsy
B. Stool leukocyte test
C. CT of the abdomen and pelvis
D. Fecal occult blood test
Explanations
(c) A. A rectal biopsy showing the absence of ganglion cells in both the submucosal and muscular layers of the
involved bowel is the most appropriate diagnostic study for Hirschsprung disease.
(u) B. Stool leukocyte testing can indicate an infectious etiology of diarrhea and is not indicated in the diagnosis of
Hirschsprung disease.
(u) C. Radiographic examination may show dilated proximal colon and absence of gas in the pelvic colon, but is not
diagnostic for Hirschsprung disease.
(u) D. Fecal occult blood testing is not indicated in the diagnosis of Hirschsprung disease.
Ref: (13)

184. Clinical Intervention/Urology/Renal


A patient has been followed for 3 years with a continual decline in glomerular filtration rate (GFR). Currently the GFR
is 10 ml/min and examination of the patient reveals a pericardial friction rub. Which of the following is the most
appropriate intervention at this time?
A. Hemodialysis
B. Continue to observe
C. Administration of high dose steroids
D. Bilateral nephrectomy
Explanations
(c) A. The patient has end stage renal disease and with the pericardial friction rub is in need of immediate
hemodialysis.
(u) B. The patient already has significant renal compromise, further observation will continue to lead to further
complications.
(u) C. High dose steroids have no benefit in end stage renal disease.
(u) D. Surgical removal of the kidneys will not change the progression of the disease.
Ref: (28)

185. Diagnosis/Endocrinology
An adult male is noted to have a hard nodule in the helix of his left ear that discharges a small amount of chalky,
white crystals. He denies any history of trauma. Which of the following is the most likely diagnosis?
A. Rheumatoid nodule
B. Keloid
C. Cutaneous cyst
D. Tophus
Explanations
(u) A. A rheumatoid nodule does not discharge any material unless subjected to repetitive trauma and tend to be
wide spread over the body as well.
(u) B. The lack of trauma history and the discharge make keloid unlikely.
(u) C. A cutaneous cyst will generally be smooth and may have a comedone associated with it. Any discharge tends
to be a foul smelling sebaceous material.
(c) D. The discharge of crystalline material is typical for a tophus lesion which represent underlying gout.
Ref: (3)

70
186. Diagnosis/Pulmonology
Which histologic type of lung cancer is typically centrally located?
A. Adenocarcinoma
B. Bronchoalveolar
C. Large cell
D. Squamous cell
Explanations
(u) A. Adenocarcinoma of the lung typically presents as a peripheral lesion.
(u) B. Bronchoalveolar carcinoma, actually a subset of adenocarcinoma of the lung, typically presents as a peripheral
lesion.
(u) C. Large cell lung cancers usually develop as peripheral lesions.
(c) D. Most squamous cell lung cancers are centrally located.
Ref: (28)

187. Clinical Therapeutics/Obstetrics/Gynecology


A 25 year-old G2P2 married female presents to the clinic for birth control counseling. Her past history includes deep
vein thrombosis with her last pregnancy. She does not want another pregnancy for at least 4 years. The birth control
method that would be best for this patient is which of the following?
A. Vaginal ring
B. Transdermal patch
C. Combined oral contraceptive pill
D. Levonorgestrel releasing IUD
Explanations
(u) A. The risk of venous thromboembolism with combined hormone in ring is similar to the oral contraceptive pill
(OCP).
(u) B. The risk of venous thromboembolism with combined hormone in patch is similar to OCP.
(u) C. The use of oral OCPs triples a user's risk of venous thromboembolism.
(c) D. IUD releases potent progestin only; is not a combination contraceptive.

Ref: (8)

188. History & Physical/Orthopedics/Rheumatology


Which of the following is an extra-articular manifestation of rheumatoid arthritis?
A. Vasculitis
B. Malar rash
C. Coronary artery aneurysms
D. Periorbital xanthelasma
Explanations
(c) A. Vasculitis affecting any organ system is seen in patients with severe rheumatoid arthritis.
(u) B. A malar rash is frequently seen in systemic or discoid lupus.
(u) C. Coronary artery aneurysms are frequently a manifestation of Kawasaki's disease.
(u) D. Xanthelasma is a skin finding that occurs in hyperlipidemia.
Ref: (26)

71
189. Clinical Intervention/Hematology
A patient receiving heparin therapy for 6 days for deep vein thrombosis develops thrombosis at the IV site. The INR is
1.1. The aPTT is 66 seconds and the platelet count is 47,000 down from 148,000 on admission. Which of the
following is the most appropriate treatment?
A. Protamine sulfate administration
B. Platelet administration
C. Discontinue the heparin
D. Schedule the patient for plasmapheresis
Explanations
(u) A. See C for explanation.
(u) B. See C for explanation.
(c) C. Once the diagnosis of heparin induced thrombocytopenia is suspected treatment should include immediate
discontinuation of all forms of heparin and treatment with a direct thrombin inhibitor should begin. Platelet
administration is generally not indicated. Protamine sulfate is used for heparin overdosage and plasmapheresis
would not be indicated as the platelet count will resume when the heparin is discontinued.
(u) D. See C for explanation.
Ref: (28)

190. Diagnostic Studies/Cardiology


A 76 year-old male presents after returning from a Safari in Africa. Seven days ago he experienced chest pressure
lasting one hour that did not respond to three sublingual nitroglycerin tablets. There was no ability to have lab work or
an EKG. The pain has not returned. If the patient had a non-STEMI myocardial infarction, which of the following
studies will still be positive?
A. Electrocardiogram
B. Myoglobulin
C. CK-MB index
D. Troponin I
Explanations
(u) A. Patients suffering from a non-STEMI myocardial infarction will not develop Q waves and most likely will have a
normal EKG five days after an acute event.
(u) B. Myoglobulin is a nonspecific enzyme that is released into the circulation after any skeletal muscle damage,
including a myocardial infarction. It is the first enzyme that becomes positive in the setting of acute myocardial
infarction but its non-specific measurement makes it less useful in the setting of acute myocardial infarction. It
returns to baseline within 24 hours after infarction.
(u) C. CK-MB index has improved sensitivity for myocardial muscle damage that occurs with acute myocardial
infarction but it returns to baseline within 2-3 days after injury.
(c) D. Troponin I levels will stay positive for at least one week following myocardial infarction and is the preferred
enzyme to measure in this setting.
Ref: (28)

191. Diagnosis/ENT/Ophthalmology
A 75 year-old patient with history of macular degeneration and hypertension presents with complaint of sudden onset
of visual loss in the left eye. The patient denies pain. On examination you note a dome-shaped retina and subretinal
fluid that shifts with position changes. Which of the following is the most likely diagnosis in this patient?
A. Central retinal vein occlusion
B. Acute angle-closure glaucoma
C. Acute nongranulomatous anterior uveitis
D. Serous retinal detachment
Explanations
(u) A. Central retinal vein occlusion is characterized by sudden monocular visual loss on examination there would be
disc swelling, venous engorgement, cotton-wool spots, and diffuse retinal hemorrhages.
(u) B. Acute angle-closure glaucoma is characterized by pain and blurred vision. On examination the eye is red, the
cornea is steamy, and the pupil is moderately dilated and nonreactive to light.

72
(u) C. Acute nongranulomatous anterior uveitis presents with acute unilateral eye pain, redness, photophobia, and
vision loss.
(c) D. Serous retinal detachment is characterized by a dome shaped retina and subretinal fluid that shifts position
with posture changes. Serous retinal detachment results from subretinal fluid accumulation which can occur in
exudative age-related macular degeneration.
Ref: (28)

192. Scientific Concepts/Psychiatry/Behavioral Medicine


A patient with known drug dependence mentions that he commonly sees sounds and hears colors. What is his drug
of choice?
A. Lysergic acid diethylamide
B. Cannabis sativa
C. Cocaine
D. Heroin
Explanations
(c) A. Lysergic acid diethylamide is LSD, the phenomenon that this patient is experiencing is called synesthesia,
which is common in patients who abuse hallucinogens.
(u) B. See A for explanation.
(u) C. See A for explanation.
(u) D. See A for explanation.
Ref: (14)

193. Clinical Therapeutics/Infectious Diseases


A disulfiram-like reaction may occur when alcohol is consumed with which of the following antibiotics?
A. Ampicillin
B. Levofloxacin (Levaquin)
C. Metronidazole (Flagyl)
D. Erythromycin
Explanations
(u) A. See C for explanation.
(u) B. See C for explanation.
(c) C. Agents that may elicit a disulfiram-like reaction include calcium carbamide, hypoglycemic sulfonylureas,
chloramphenicol, furazolidone, metronidazole, quinacrine, and certain cephalosporins.
(u) D. See C for explanation.
Ref: (15)

194. History & Physical/Pulmonology


Which of the following is the most likely presentation of an acute pulmonary embolism (PE) in a patient without
preexisting cardiac or pulmonary disease?
A. Anginal chest pain
B. Cough
C. Tachypnea
D. Palpitations
Explanations
(u) A. While it is quite common for PE to present with pleuritic chest pain, angina-like pain is only rarely reported.
(u) B. Cough is reported in roughly one-third of patients with PE. But is not the most common presentation.
(c) C. Tachypnea is the most common symptom in acute PE.
(u) D. Palpitations are uncommonly reported in acute PE.
Ref: (7)

73
195. Diagnostic Studies/Obstetrics/Gynecology
A 24 year-old sexually active woman presents to the clinic complaining of dysmenorrhea, dyspareunia and backache
that occurs premenstrually. Pelvic examination shows pain upon uterine motion and uterosacral nodularity in the
posterior vaginal fornix. The definitive diagnosis for this patient requires which of the following?
A. Pelvic ultrasound
B. CT of the abdomen
C. Laparoscopy with biopsy
D. CA-125 test
Explanations
(u) A. Pelvic ultrasound has little value in diagnosing endometriosis.
(u) B. CT of the abdomen has little value in diagnosing endometriosis.
(c) C. Diagnosis of endometriosis must be made by direct visualization. Laparoscopy with biopsy is the most
appropriate diagnostic study in this patient.
(u) D. CA-125 may be elevated in endometriosis, but it is not diagnostic.
Ref: (8)

196. Clinical Intervention/Neurology


A 45 year-old male patient presents to the office with a complaint of dizziness and vertigo that occurs suddenly when
he rolls out of bed in the morning. He denies previous illness or any medical problems. On physical exam you note
the presence of lateral nystagmus after a few second latency period. What is your recommendation for this patient?
A. Low dose diazepam
B. Repositioning maneuvers
C. Diuretics and a low salt diet
D. MRI of the brain
Explanations
(u) A. Low dose diazepam is used to treat acute vertigo.
(c) B. This patient has benign paroxysmal positional vertigo (BPPV) and repositioning maneuvers are
recommended to move endolymphatic debris out of the posterior semicircular canal.
(u) C. Diuretics and a low salt diet are used to treat Meniere's disease.
(u) D. An MRI of the brain should be ordered in a patient in whom a central etiology of vertigo is suspected.
Ref: (1)

197. Clinical Therapeutics/Dermatology


A patient presents with 3 weeks of worsening pruritic rash located on the upper extremities and interdigital spaces.
He is a migrant farm worker with no history of skin disorders. Examination reveals excoriated, erythematous papules
with numerous 3-4 mm long, narrow tracts spreading from the papules. What is the best treatment for this patient?
A. Permethrin (Nix)
B. Prednisone (Deltasone)
C. Montelukast (Singulair)
D. Pimecrolimus (Elidel)
Explanations
(c) A. Permethrin is used to kill the scabies mite.
(u) B. Corticosteroids have no affect on mites.
(u) C. Montelukast would possibly provide symptomatic relief but would not be curative for scabies.
(u) D. Pimecrolimus has no affect on destruction of mites.
Ref: (28)

74
198. Diagnosis/Gastrointestinal/Nutritional
A 32 year-old male with history of tobacco abuse presents with an intermittent burning sensation in his chest for six
months, worsening over the past 2 weeks. His wife has noticed episodes of coughing at night. He denies dysphagia,
weight loss, hematemesis, or melena. His vital signs are all normal and physical examination is unremarkable. Which
of the following is the most likely diagnosis?
A. Peptic ulcer disease
B. Acute gastritis
C. Gastroesophageal reflux disease
D. Esophageal stricture
Explanations
(u) A. Peptic ulcer disease typically presents with midepigastric pain and is not usually associated with cough.
(u) B. Acute gastritis would have a duration of less than 2 weeks.
(c) C. Gastroesophageal reflux disease presents with at least weekly episodes of heartburn and typically occurs
after meals and upon reclining. Patients may complain of regurgitation, chronic cough, laryngitis, or sore
throat.
(u) D. Esophageal stricture develops from long term gastroesophageal reflux disease. Patients with esophageal
stricture usually have dysphagia.
Ref: (1)

199. Clinical Intervention/Cardiology


A 52 year-old patient with episodes of syncope has an electrocardiogram which shows a consistently prolonged PR
interval with a missing QRS every two beats. Which of the following is the most effective management?
A. Permanent pacing
B. Beta-blocker
C. ACE Inhibitor
D. Defibrillation
Explanations
(c) A. This is consistent with ECG findings of a Mobitz type II AV block. Since the patient is symptomatic this type of
AV block requires a permanent pacing to prevent total AV disassociation.
(u) B. Beta-blockers will slow conduction from the AV node and is not indicated with this type of AV block.
(u) C. There is no indication for ACE Inhibitors in Mobitz Type II heart block.
(u) D. Defibrillation is not indicated in a person with AV block.
Ref: (7)

200. Health Maintenance/Orthopedics/Rheumatology


Which of the following groups is most likely to present with Duchenne's muscular dystrophy?
A. Adolescent females
B. Middle-aged males
C. Infant females
D. Toddler-aged males
Explanations
(u) A. See D for explanation.
(u) B. See D for explanation.
(u) C. See D for explanation.
(c) D. Duchenne's muscular dystrophy, a genetic defect on the short arm of the X chromosome, affects toddler-aged
males.
Ref: (28)

75
201. Diagnostic Studies/Pulmonology
What is the diagnostic modality of choice to diagnose cystic fibrosis (CF)?
A. Chest radiograph
B. Clinical features
C. Sweat chloride concentration testing
D. Genotyping
Explanations
(u) A. Radiographic findings may suggest the diagnosis but are not specific.
(u) B. While clinical features may suggest the need for testing they are not useful in confirming the diagnosis.
(c) C. The standard for diagnosis is two positive sweat chloride concentration tests obtained on separate days or
identification of CF mutations or an abnormal nasal potential difference measurement.
(u) D. Genotyping screens for only a fraction of the known CF mutations.
Ref: (28)

202. Scientific Concepts/Obstetrics/Gynecology


A pregnant 28 year-old female presents at 30 weeks gestation complaining of fatigue and headache. Her vital signs
and physical examination are normal. Routine hemoglobin screening is 10.1 g/dL. Peripheral smear shows
microcytic, hypochromic red blood cells. Besides the physiologic dilution of pregnancy, what type of anemia is most
likely in this patient?
A. Iron deficiency
B. Folic acid deficiency
C. Thalassemia
D. Hereditary spherocytosis
Explanations
(c) A. Iron deficiency anemia is responsible for 95% of anemias during pregnancy.
(u) B. Folic acid deficiency anemia is common where nutrition is inadequate.
(u) C. Thalassemia is a rare cause of anemia in pregnancy.
(u) D. Hereditary spherocytosis is rare during pregnancy.
Ref: (8)

203. Diagnosis/Neurology
A 70 year-old male is brought to the office by his wife. She is complaining that her husband has been having great
difficulty remembering things, however remains alert. She also complains that he is having difficulty getting to the
bathroom on time. On physical examination you notice that the patient's gait is wide-based and slow and he walks
without lifting his feet off the floor. You also notice a tremor in his right hand. What is the most likely diagnosis?
A. Lewy body dementia
B. Parkinson's disease
C. Normal pressure hydrocephalus
D. Frontotemporal dementia
Explanations
(u) A. Patients with Lewy body dementia have hallucination, Parkinsonism, fluctuating alertness, and falls.
(c) B. Patients with Parkinson's disease have clinical features such as bradykinesia, rest tremor, rigidity, postural
instability, autonomic dysfunction and behavioral changes. Of the diseases listed, Parkinson's is the most
common.
(u) C. Patients with normal pressure hydrocephalus have an abnormal gait (apraxic or ataxic), dementia, and urinary
incontinence.
(u) D. Patients with frontotemporal dementia have predominant behavioral symptoms, dementia, apraxia,
Parkinsonism, and motor neuron disease.
Ref: (1)

76
204. Clinical Therapeutics/Psychiatry/Behavioral Medicine
Higher doses of SSRIs are usually required in which of the following conditions?
A. Obsessive-compulsive disorders
B. Depression
C. Manic depression
D. Panic disorder
Explanations
(c) A. Higher doses of SSRIs are needed in the treatment of OCD for a beneficial effect.
(u) B. Although patients with depression may occasionally need to have an increase in the SSRI dosage, higher
doses are not needed for beneficial effect.
(u) C. Patients who are bipolar will often have worsening of their manic symptoms with the use of SSRIs.
(u) D. SSRIs are good drugs to treat patients with panic disorder, however, these patients are particularly sensitive to
the activating effects of SSRIs, so they should be started initially at small dosages and titrated up slowly.
Ref: (14)

205. Health Maintenance/Cardiology


According to the Joint National Commission VII Guidelines, blood pressure targets are lower in patients with diabetes
mellitus and what other condition?
A. Liver disease
B. Renal disease
C. Thyroid disease
D. Peripheral vascular disease
Explanations
(u) A. See B for explanation.
(c) B. Blood pressure targets for hypertensive patients at the greatest risk for cardiovascular events, particularly
those with diabetes and chronic kidney disease, are lower (less than 130/80) than for those individuals with
lower cardiovascular risk (goal is less than 140/90).
(u) C. See B for explanation.
(u) D. See B for explanation.
Ref: (28)

206. Clinical Intervention/Pulmonology


A 67 year-old man presents complaining of gradually worsening fatigue and shortness of breath. He is a previous
smoker with an 80 pack-year smoking history. He denies chest pain, night sweats, or hemoptysis. On physical
examination, you note a very thin male who appears older than his stated age. Lung and heart sounds are barely
audible to auscultation. Which of the following interventions is likely to alter the disease course?
A. Inhaled bronchodilator therapy
B. Inhaled steroid therapy
C. Home oxygen
D. Theophylline
Explanations
(u) A. Inhaled bronchodilators afford symptomatic relief for some patients with COPD but do not alter the disease
course.
(u) B. Inhaled steroid therapy may reduce the number and severity of COPD exacerbations but has not been
shown to alter the disease course.
(c) C. Home oxygen therapy has been shown to prolong life in patients with COPD and alter the natural history of
the disease.
(u) D. Theophylline is a third-line agent for treating COPD and will not alter the natural history of the disease
Ref: (28)

77
207. History & Physical/Gastrointestinal/Nutritional
An afebrile 22 year-old female presents with 2 days of a painful rectal mass which worsens with defecation. She
denies any blood with bowel movements. She admits to ongoing constipation and generally poor diet of fast food.
She is otherwise healthy. On examination there is a tender, swollen, bluish, ovoid mass visible at the anal margin.
Which of the following is the most likely diagnosis?
A. Rectal prolapse
B. Thrombosed hemorrhoid
C. Internal Hemorrhoid
D. Rectal polyp
Explanations
(u) A. Rectal prolapse shows radiating folds and is typically painless.
(c) B. A thrombosed hemorrhoid causes acute pain increased with defecation and sitting.
(u) C. Internal hemorrhoids are an enlargement of the normal vascular cushions located above the pectinate line.
They are not usually palpable and might cause bright red bleeding during defecation.
(u) D. Polyps are soft and may be difficult or impossible to feel. Proctoscopy is usually required for diagnosis.
Ref: (3)

208. Clinical Therapeutics/Urology/Renal


A 26 year-old sexually active woman has a 3-day history of dysuria, frequency, and urgency. She has a fever of 102
degrees F orally with shaking chills and right-sided costovertebral angle tenderness. Urinalysis reveals 10-20
RBCs/HPF, 30 WBCs in clumps/HPF, 3-4 WBC casts and 3+ bacteria. Which of the following is the treatment of
choice for outpatient management?
A. Ciprofloxacin
B. Erythromycin
C. Doxycycline
D. Amoxicillin
Explanations
(c) A. Treatment with a quinolone is first line treatment of pyelonephritis on an outpatient basis. If the patient were
admitted then empiric treatment with Ampicillin and an aminoglycoside IV would be initiated until culture and
sensitivity results were obtained.
(u) B. See A for explanation.
(u) C. See A for explanation.
(u) D. See A for explanation.
Ref: (28)

209. Scientific Concepts/ENT/Ophthalmology


Which of the following is the most common etiologic agent associated with acute bacterial sinusitis in the adult
population?
A. Staphylococcus aureus
B. Streptococcus pneumoniae
C. Pseudomonas aeruginosa
D. Mycoplasma pneumoniae
Explanations
(u) A. See B for explanation.
(c) B. The typical pathogens associated with acute bacterial sinusitis are Streptococcus pneumoniae, other
streptococci species, and Haemophilus influenzae. Moraxella catarrhalis and Staphylococcus aureus are less
common causes. Pseudomonas is a less common cause and would be associated with nosocomial infection
often in a critically ill patient. Mycoplasma is not a typical cause of bacterial sinusitis.
(u) C. See B for explanation.
(u) D. See B for explanation.
Ref: (28)

78
210. Diagnosis/Cardiology
A patient presents to the office following a syncopal episode. The patient claims that the syncope occurs when he
changes position such as rolling over in bed or when he bends over to tie his shoes. Which of the following is the
most likely explanation for this presentation?
A. Carotid sinus hypersensitivity
B. Vasovagal episode
C. Subclavian steal syndrome
D. Atrial myxoma
Explanations
(u) A. Carotid sinus hypersensitivity may present with syncope but is usually related to tight collars or when
excessively turning the head.
(u) B. Vasovagal episodes may occur with syncope as its manifestation but it is not caused by changes in position.
(u) C. Subclavian steal syndrome may present with syncope that is related to exercise of the affected arm which
results in a decreased pulse when the Adson maneuver is performed.
(c) D. Atrial myxoma most commonly presents with sudden onset of symptoms that are typically positional in nature
due to the effect that gravity has on the tumor. Myxomas are the most common type of primary cardiac tumor
in all age groups and are most commonly found in the atria.
Ref: (28)

211. Diagnostic Studies/Orthopedics/Rheumatology


Spina bifida occulta is usually detected by which initial diagnostic evaluation?
A. Electroencephalogram
B. Alpha-fetoprotein levels
C. Folic acid levels
D. X-ray of the spine
Explanations
(u) A. Electroencephalogram (EEG) cannot detect or diagnose spina bifida occulta.
(u) B. Alpha-fetoprotein, measured at 16-18 weeks of pregnancy, if elevated, would indicate a neural tube defect,
such as spina bifida.
(u) C. Folic acid has been shown to decrease the incidence of neural tube defects, not as an aid in diagnosis of the
defect.
(c) D. X-ray or MRI is the definitive test to diagnose spina bifida occulta, showing the vertebral bony defect.
Ref: (5)

212. Scientific Concepts/Hematology


Which of the following conditions is caused by a disorder in the red blood cell membrane?
A. G6PD deficiency
B. Folate deficiency
C. Sickle cell anemia
D. Hereditary spherocytosis
Explanations
(u) A. G6PD deficiency results in an increased destruction of red cells that leads to anemia.
(u) B. Folate is essential in DNA synthesis, lack of folate leads to ineffective erythropoiesis.
(u) C. Sickle cell anemia is a hemoglobinopathy which leads to increased red cell destruction.
(c) D. The cell membrane defect of spherocytosis leads to hemolysis due to trapping of the cells by the spleen.
Ref: (28)

79
213. Clinical Therapeutics/ENT/Ophthalmology
A patient presents complaining of right ear pain and itching. On physical examination you note erythema and edema
of the right ear canal with purulent exudate. Palpation of the tragus and manipulation of the auricle is painful. The
tympanic membrane is not well visualized. Which of the following is the treatment of choice for this patient?
A. Amoxicillin
B. Aqueous irrigation
C. Tympanocentesis
D. Polymyxin B sulfate
Explanations
(u) A. Amoxicillin is the initial treatment of choice in acute otitis media, not otitis externa.
(u) B. Aqueous irrigation is not indicated in the treatment of otitis externa.
(u) C. Tympanocentesis is not indicated in the treatment of otitis externa.
(c) D. Polymyxin B sulfate is the treatment of choice for otitis externa.
Ref: (28)

214. Health Maintenance/Obstetrics/Gynecology


A 56 year-old female presents to the clinic wanting testing for ovarian cancer. Her best friend was just diagnosed with
Stage 4 primary cancer of the ovary. She denies family history of breast or ovarian cancer. According to screening
guidelines, which of the following do you recommend?
A. BRCA 1-2 gene testing
B. A pelvic ultrasound yearly
C. Continue her yearly women's health examinations
D. CA-125 testing
Explanations
(u) A. BRCA 1-2 has been found to be associated with ovarian cancers in approximately 5% of cases. However the
expression of this gene in BRACA 1-2 carriers are unpredictable. Therefore, routine testing for this gene has
limited value at this time.
(u) B. Refined imaging techniques offer promise for the future however much work remains before these tests are
considered accurate or cost effective screening tests.
(c) C. The patient is advised to have regular pelvic exams. The limited prevalence of ovarian cancer and the lack of
sensitivity and specificity of current available tests have so far prevented the implementation of routine ovarian
cancer screening of the general population.
(u) D. The CA-125 recognizes the antigen CA-125 which is present in serous ovarian tumors but not in mucinous or
nonepithelial ovarian tumors however the test is not considered a cost effective screening test.
Ref: (8)

215. Clinical Therapeutics/Cardiology


Which of the following is the optimal therapy for a 76 year-old patient with no allergies who has chronic atrial
fibrillation?
A. Aspirin
B. Clopidogrel (Plavix)
C. Warfarin (Coumadin)
D. Low molecular weight heparin
Explanations
(u) A. Aspirin's role to prevent thromboembolism in atrial fibrillation is limited to patients with no risk factors who are
under age 65.
(u) B. Clopidogrel is not the optimal therapy for patients with atrial fibrillation.
(c) C. Patients older than age 75 who have chronic atrial fibrillation should be anticoagulated with warfarin to
maintain an INR between 2.5 and 3.0 for optimum therapy unless a contraindication to therapy exists.
(u) D. Due to the increased costs and need for parenteral therapy, daily subcutaneous heparin is not first line therapy
unless warfarin therapy is contraindicated.
Ref: (7)

80
216. Scientific Concepts/Pulmonology
Which of the following causes of pneumonia is most likely to be complicated by diarrhea?
A. Legionella
B. Chlamydophila
C. Mycoplasma
D. Pneumococcal
Explanations
(c) A. Legionella is a water-borne pathogen that can cause diarrhea as a complication of its infection.
(u) B. Chlamydophila pneumonia is associated with dry cough, low-grade fever, and hoarseness.
(u) C. Mycoplasma pneumonia is most likely complicated by bullous myringitis, hoarseness, rash, and chronic cough.
(u) D. Pneumococcal pneumonia presents with a single rigor, rust-colored sputum, and productive cough. There is no
diarrhea as part of its infection.
Ref: (7)

217. Diagnosis/Psychiatry/Behavioral Medicine


A 38 year-old female is brought to the emergency room with a complaint of paralysis of her arms bilaterally. Her
history is significant for a sexual assault two days previously. Her physical examination is unremarkable with normal
reflexes. There is no apparent physical explanation for her paralysis. What is the most likely diagnosis?
A. Body dysmorphic disorder
B. Conversion disorder
C. Hypochondriasis
D. Factitious disorder
Explanations
(u) A. Body dysmorphic disorder patients have a preoccupation with an imagined defect in appearance that
causes significant distress.
(c) B. Patients have abrupt loss of motor or sensory function caused by psychological factors that often precede
the physical symptoms
(u) C. Hypochondriasis is characterized by six months or more of general, non delusional preoccupation with fears
of having a serious disease.
(u) D. Patients with factitious disorder fake illness.
Ref: (14)

218. Health Maintenance/ENT/Ophthalmology


At what age should a child's eyes be consistently well-aligned?
A. Two months
B. Six months
C. Twelve months
D. Eighteen months
Explanations
(u) A. See B for explanation.
(c) B. A child's eyes should be consistently well-aligned by five to six months of age.
(u) C. See B for explanation.
(u) D. See B for explanation.
Ref: (13)

81
219. Clinical Intervention/Cardiology
Patients who undergo percutaneous angioplasty or who have coronary artery revascularization often are treated with
glycoprotein IIb/IIIa inhibitors. What is the major side effect associated with these agents?
A. Hypotension
B. Bleeding
C. Coronary vasospasm
D. Acute renal failure
Explanations
(u) A. See B for explanation.
(c) B. Glycoprotein IIb/IIa inhibitors have their activity in the final stages of platelet bridging and are associated with
bleeding when used in the management of acute myocardial infarction. Since they are effective at treating and
preventing new clot formation, bleeding is the main concern and complication with the use of these agents.
(u) C. See B for explanation.
(u) D. See B for explanation.
Ref: (7)

220. Clinical Therapeutics/Gastrointestinal/Nutritional


A cirrhotic patient presents with progressive drowsiness and delirium. Physical examination reveals asterixis and
tremor. Which of the following medications would you initiate?
A. Empiric antibiotic therapy
B. Lactulose (Kristalose)
C. Bolus fluid challenge with sodium chloride
D. Hydrochlorothiazide (Diuril)
Explanations
(u) A. See B for explanation.
(c) B. This patient has hepatic encephalopathy. Increased ammonia levels contribute to the mental status changes
associated with hepatic encephalopathy. Lactulose leads to a change in bowel flora so that fewer ammonia
forming organisms are present and also helps in the acidification of colon contents which leads to a
nonabsorbable ammonium ion creation.
(u) C. See B for explanation.
(u) D. See B for explanation.
Ref: (1)

221. Diagnostic Studies/Infectious Diseases


A 19 year-old college student presents to the student health center complaining of a sore throat and mild flu-like
symptoms for 2 days. A rapid strep screen and culture are negative. Symptomatic therapy is instituted. The patient
returns 3 weeks later complaining of continued sore throat with increasing fatigue and onset of fever. Physical
examination reveals an exudative pharyngitis, tender enlarged posterior cervical lymphadenopathy, and the spleen is
palpable. Which of the following is most appropriate as the next step in diagnosis?
A. Mono spot
B. Repeat strep screen
C. Anti-EBV antibodies
D. Lymph node biopsy
Explanations
(c) A. With the presence of exudative pharyngitis associated with posterior cervical lymphadenopathy and
splenomegaly, infectious mononucleosis is the most likely diagnosis. A mono spot would most likely be
positive since the patient has had symptoms for more than 3 weeks.
(u) B. A strep screen tests for the presence of beta-hemolytic Streptococcus group A, which would not account for
the splenomegaly.
(u) C. Anti-EBV antibodies measure specific antibody titers and are not a cost effective diagnostic test at this time.
(h) D. Lymph node biopsy is not indicated and may lead to complications in the setting of acute mononucleosis.
Ref: (28)

82
222. History & Physical/ENT/Ophthalmology
A 50 year-old patient presents complaining of headache and left eye pain for 5 hours. The patient admits to
decreased vision in the left eye. The patient also complains of associated nausea. Which of the following is most
likely on physical examination?
A. Neovascularization
B. Central vision loss
C. Impaired red reflex
D. Moderately dilated, nonreactive pupil
Explanations
(u) A. Neovascularization is seen with diabetic retinopathy not glaucoma.
(u) B. Central vision loss is seen with macular degeneration not glaucoma.
(u) C. Impaired red reflex is common with cataract not glaucoma.
(c) D. The pupil is moderately dilated and nonreactive to light in acute angle closure glaucoma.
Ref: (28)

223. Health Maintenance/Cardiology


Which of the following is an absolute contraindication for the performance of exercise stress testing for patients who
wish to start an exercise program?
A. Second degree heart block type 1
B. Severe aortic stenosis
C. Atrial fibrillation with controlled ventricular response
D. Recent diagnosis of lung cancer
Explanations
(u) A. See B for explanation.
(c) B. Contraindications to stress testing include rest angina within the last 48 hours, unstable cardiac rhythm,
hemodynamically unstable patient, severe aortic stenosis, acute myocarditis, uncontrolled heart failure, and
active infective endocarditis.
(u) C. See B for explanation.
(u) D. See B for explanation.
Ref: (7)

224. Diagnosis/Orthopedics/Rheumatology
A mother brings her 14 month-old son to your clinic. Earlier today she lifted her son by grabbing him by the wrists and
pulling him up off the floor. The child is sitting in his mother's lap with his left forearm is extended and in pronation. He
is refusing to move the left arm, forearm or wrist. The arm and joints appear normal with no noted deformities, edema
or erythema. Distal pulses and capillary refill are normal and he can move his fingers. Which of the following is the
most likely diagnosis?
A. Nursemaid's elbow
B. Fractured left wrist
C. Osteochondritis dissecans
D. Child abuse
Explanations
(c) A. This clinical history is classic for radial head dislocation or nursemaid's elbow.
(u) B. There is no edema, deformity or erythema to suggest a fractured wrist.
(u) C. Osteochondritis dissecans is avascular necrosis of subchondral bone, most commonly seen during
adolescence.
(a) D. Although child abuse could be suspected the clinical history best describes nursemaid's elbow.
Ref: (29)

83
225. Clinical Therapeutics/Endocrinology
A patient with Type 2 diabetes uses a mixture of NPH and regular insulin twice daily. She consistently has mid-
afternoon bouts of hypoglycemia, despite eating her meals as scheduled. Which modification is most appropriate to
reduce her mid-afternoon hypoglycemic events?
A. Increase carbohydrate content of lunchtime meal
B. Increase fat content of breakfast meal
C. Reduce morning dose of NPH insulin
D. Reduce morning dose of regular insulin
Explanations
(u) A. Increasing carbohydrate content of lunchtime meal would be inappropriate if the glycemic control is
otherwise normal; might promote weight gain.
(u) B. Increasing fat content of breakfast meal would not aid in symptoms as it would slow glucose absorption and
potentially worsen the hypoglycemia.
(c) C. Reducing the morning dose of NPH insulin would be most appropriate as it exerts its greatest effect on the
noontime meal.
(u) D. Reducing the morning dose of regular insulin would have more effect on noon blood sugar control.
Ref: (28)

84
Physician Assistant Education Association
PACKRAT
Comment Form

Please fill out this form and return it to: Physician Assistant Education Association
300 N. Washington St., Ste 505
Alexandria, VA 22314

YES NO

1. I found the PACKRAT a useful study tool.

Comment:

2. I was able to understand the directions in the Directions and Explanations.

Comment:

3. The feedback provided was easy to understand.

Comment:

4. I know where my weaknesses lie and where to focus my studies.

Comment:

5. The content of the examination was comprehensive.

Comment:

85
6. The examination questions were clearly worded.

Comment:

7. Suggestions for improvement:

_________________________________________________________________

_________________________________________________________________

_________________________________________________________________

_________________________________________________________________

86

You might also like